Этого треда уже нет.
Это копия, сохраненная 18 апреля 2016 года.

Скачать тред: только с превью, с превью и прикрепленными файлами.
Второй вариант может долго скачиваться. Файлы будут только в живых или недавно утонувших тредах. Подробнее

Если вам полезен архив М.Двача, пожертвуйте на оплату сервера.
180 Кб, 815x508
115 Кб, 900x900
Математика, тред 45 #343958 В конец треда | Веб
Сперва нужно понятным образом объяснить аксиоматический метод.
То есть рассказать, что такое "система вещей" по Гильберту, что такое аксиомы.

Далее нужно ввести элементы логики.
Объяснить, что такое предикат, что такое функциональный символ, что такое квантор и т.д.

Теория множеств вводится так.

Существует система вещей, называемых множествами.
Для них задано логическое отношение "быть элементом".
Всякая вещь A либо является элементом вещи B, либо не является.

Кроме того, для этих вещей задано отношение равенства с традиционными свойствами:
рефлексивность, симметричность, транзитивность, подстановочность.

Далее нужно дать несколько определений.

1. Множество называется пустым, если никакое множество не является его элементом.

2. Множество называется индуктивным, если хотя бы одно пустое множество является его элементом и если оно вместе с каждым элементом x содержит элемент {x}.

3. A называется подмножеством B, если всякий элемент A является элементом B.
В таком случае B называется надмножеством A.

4. Вместо того, чтобы говорить "множество множеств", мы часто будем говорить "семейство множеств".

5. Пусть M - семейство. A называется объединением M, если каждое множество из M - подмножество A и каждый элемент A - элемент хотя бы одного элемента M.

6. Семейство называется регулярным, если в нём есть такое множество, что каждый его элемент не входит в семейство.

7. Непустое C называется неупорядоченной парой A и B, если каждый элемент C равен либо A, либо B.

8. Пусть A и M множества.
Пусть каждое подмножество A является элементом M.
Пусть каждый элемент M является подмножеством A.
Тогда M называется "булеан A" или "множество всех подмножеств A".

9. Пусть M - множество, P - высказывание о его элементах, N - подмножество M.
Мы говорим, что N удовлетворяет P, если для всякого n из N высказывание P(n) истинно.

10. Пусть, в тех же обозначениях, N удовлетворяет P.
Пусть любое подмножество M, которое удовлетворяет P, является подмножеством N.
Мы говорим, что P выделяет множество N из множества M.
Или что N выделено предикатом P из M.

11. Пусть f - функциональный символ от двух аргументов.
Если вещь можно подставить в этот символ в качестве первого аргумента, то мы говорим, что функциональный символ осмыслен на этой вещи.
Мы говорим, что функциональный символ корректен на множестве M, если функциональный символ осмыслен на каждом элементе M.
Мы говорим, что функциональный символ является истинным функциональным высказыванием об M, если он корректен на M.

12. Мы говорим, что функциональный символ f, корректный на M, преобразует множество M во множество N, если
а) для всякого m из M существует единственный n из N такой, что f(m, n) истинно
б) для всякого n из N существует элемент m из M такой, что f(m, n) истинно

13. Пусть есть функциональный символ, корректный на M. Пусть существует такое N, что f преобразует M в N.
Тогда мы говорим, что f является преобразованием M.

Далее нужно сказать, что вещи нашей системы подчиняются следующим аксиомам (Цермело-Френкеля с выбором).

1. Два множества равны тогда и только тогда, когда первое является подмножеством второго и второе является подмножеством первого.

2. Для всякого множества всякое высказывание о его элементах выделяет подмножество.

3. Существует пустое множество.

4. Существует индуктивное множество.

5. Для любых двух множеств, не обязательно разных, существует неупорядоченная пара.

6. Для всякого множества существует булеан.

7. Для всякого семейства множеств существует объединение.

8. Всякое истинное функциональное высказывание о множестве является преобразованием множества.

9. Всякое непустое семейство регулярно.

10. Для всякого семейства непустых множеств существует множество, содержащее ровно по одному элементу из каждого множества семейства.

Прошлый: >>340433 (OP)
#2 #343960
>>343958 (OP)
1. https://arhivach.org/thread/18638/
2. https://arhivach.org/thread/27246/
3. https://arhivach.org/thread/27696/
4. https://arhivach.org/thread/38709/
5. https://arhivach.org/thread/46502/
6. https://arhivach.org/thread/48852/
7. https://arhivach.org/thread/52165/
8. https://arhivach.org/thread/56479/
9. https://arhivach.org/thread/63306/
10. https://arhivach.org/thread/70618/
11. https://arhivach.org/thread/74342/
12. https://arhivach.org/thread/74341/
13v1. https://arhivach.org/thread/76561/
13v2. https://arhivach.org/thread/92428/
14. https://arhivach.org/thread/78408/
15. https://arhivach.org/thread/79152/
16. https://arhivach.org/thread/82499/
17. https://arhivach.org/thread/92427/
18. https://arhivach.org/thread/84722/
19. https://arhivach.org/thread/87923/
20. https://arhivach.org/thread/91329/
21. http://arhivach.org/thread/93067/
22. https://arhivach.org/thread/94240/
23. https://arhivach.org/thread/95680/
24. https://arhivach.org/thread/96720/
25. https://arhivach.org/thread/99481/
26. https://arhivach.org/thread/100880/
27. https://arhivach.org/thread/101335/
28. http://arhivach.org/thread/106743/
29. https://arhivach.org/thread/109198/
30. https://arhivach.org/thread/114111/
31. https://arhivach.org/thread/116099/
32. https://arhivach.org/thread/118093/
33v1. https://arhivach.org/thread/122613/
33v2. https://arhivach.org/thread/122615/
34. https://arhivach.org/thread/123717/
35. https://arhivach.org/thread/128822/
36. https://arhivach.org/thread/129039/
37. https://arhivach.org/thread/131462/
38. https://arhivach.org/thread/138362/
39. https://arhivach.org/thread/138429
40. http://arhivach.org/thread/140404/
41. https://arhivach.org/thread/142386/
42. https://arhivach.org/thread/145879/
43. https://arhivach.org/thread/146833/
44. https://arhivach.org/thread/152600/
#2 #343960
>>343958 (OP)
1. https://arhivach.org/thread/18638/
2. https://arhivach.org/thread/27246/
3. https://arhivach.org/thread/27696/
4. https://arhivach.org/thread/38709/
5. https://arhivach.org/thread/46502/
6. https://arhivach.org/thread/48852/
7. https://arhivach.org/thread/52165/
8. https://arhivach.org/thread/56479/
9. https://arhivach.org/thread/63306/
10. https://arhivach.org/thread/70618/
11. https://arhivach.org/thread/74342/
12. https://arhivach.org/thread/74341/
13v1. https://arhivach.org/thread/76561/
13v2. https://arhivach.org/thread/92428/
14. https://arhivach.org/thread/78408/
15. https://arhivach.org/thread/79152/
16. https://arhivach.org/thread/82499/
17. https://arhivach.org/thread/92427/
18. https://arhivach.org/thread/84722/
19. https://arhivach.org/thread/87923/
20. https://arhivach.org/thread/91329/
21. http://arhivach.org/thread/93067/
22. https://arhivach.org/thread/94240/
23. https://arhivach.org/thread/95680/
24. https://arhivach.org/thread/96720/
25. https://arhivach.org/thread/99481/
26. https://arhivach.org/thread/100880/
27. https://arhivach.org/thread/101335/
28. http://arhivach.org/thread/106743/
29. https://arhivach.org/thread/109198/
30. https://arhivach.org/thread/114111/
31. https://arhivach.org/thread/116099/
32. https://arhivach.org/thread/118093/
33v1. https://arhivach.org/thread/122613/
33v2. https://arhivach.org/thread/122615/
34. https://arhivach.org/thread/123717/
35. https://arhivach.org/thread/128822/
36. https://arhivach.org/thread/129039/
37. https://arhivach.org/thread/131462/
38. https://arhivach.org/thread/138362/
39. https://arhivach.org/thread/138429
40. http://arhivach.org/thread/140404/
41. https://arhivach.org/thread/142386/
42. https://arhivach.org/thread/145879/
43. https://arhivach.org/thread/146833/
44. https://arhivach.org/thread/152600/
60 Кб, 600x433
#3 #343964
Я вам суть математикохейтерства принес.
>>343968>>345173
#4 #343968
>>343964
Тян в матанотреде, все в ковалевскую
>>343972>>344179
#5 #343972
>>343968
Этой фотографии больше лет, чем некоторым школьникам из /b/.
#6 #344006
Где моё корректное определение N? Вы сосёте мой хуй.
>>344013>>344032
#7 #344011
Нам нужна новая эмпирическая наука, замена математики. Математика устарела, она стара и мертва. Мы стоим на пороге новых открытий.
>>344013>>344032
#8 #344012
Школьная математика - не математика, а хуйня. Ненужная. Каждый говёный доктор преподавания считает своим долгом написать столь же говёную книжку по элементарной математике.
>>344013>>344032
#9 #344013
>>344006
>>344011
>>344012
А что, если все эти темы выдумал один и тот же человек?
>>344014
#10 #344014
>>344013
Вроде разные. N-петух не дал определения определению, а Толстота дал.
sage #11 #344032
>>344006
>>344011
>>344012
Три столпа на которых стоит мат.тред
sage #12 #344067
Тред математики - раковая опухоль на теле /sci.
>>344069>>344178
#13 #344069
>>344067
Тред математики - ядро /sci.
#14 #344071
Алгебраическая геометрия - не математика, а хуйня. Ненужная. Каждый говёный доктор считает свои долгом написать столь же говёную книжку по алгебраической геометрии.
>>344080
#15 #344075
Да уж, убожество
#16 #344080
>>344071
Лол.
#17 #344086
Используя распределительное свойство умножения, найдите значения букв.
Вычислите:
a(2,3+1,6)=4,22,3+b

Задача за 5 класс
>>344088>>344090
sage #18 #344088
>>344086
Сложна
#20 #344092
>>344090
Ну смари правильное решение
a (2,3 + 1,6) = 4,2 2,3 + b
a 2,3 + а 1,6 = 4,2 2,3 + b
а = 4,2
b = a
1,6 = 4,2 *1,6 = 6,72

С какого хуя а = 4,2?
>>344125
45 Кб, 802x1026
#21 #344125
>>344092
Блеадь, еле разобрался.
>>344198
48 Кб, 580x401
#22 #344166
>>344168>>344481
#23 #344168
>>344166
Поясните за него.
>>344169
#24 #344169
>>344168
Он охуенен.
>>344170>>344176
#25 #344170
>>344169
Есть интервью где он рассказывает про свой дар?
>>344171
#26 #344171
>>344170
Нет.
#27 #344176
>>344169
Он лох.
#28 #344178
>>344067
Лол, это же моя фраза. Я её придумал, сучёныш!
#29 #344179
>>343968
Лол, ньюфажек, это не тян, а переодетый еврей.
#30 #344198
>>344125
у тебя нихуя не понятно воще, ты что сука чистый математик? У меня -4.6а-9.66=-9.66 и -4.6а=-19.32 от тебя.
#31 #344285
Ребят, я полный нуб в математике. В школе нихера не понимал и последняя годная оценка была в 5 классе на контрольной.
Может кто что посоветует для того чтобы как то понимать математику и уметь решать?
Спасибо
10 Кб, 264x400
#32 #344306
Кто уже читал, как оцениваете триумфальное возвращение?
#33 #344308
>>344306
Вот же человечище! С того света о гомологиях пишет.
>>344310
#34 #344310
>>344308
Кто он, блядь? Их здесь не один.
>>345853
#35 #344329
>>344306

>Les Éléments de mathématique restent, encore à ce jour, inachevés.


Лол, видимо, будет ещё продолжение.

>Springer


А вот это плохо. У них говённое качество принта.
22 Кб, 224x360
#36 #344335
Знакомьтесь, ребята, на пике - лох. Презирайте его.
>>344338
#37 #344338
>>344335
Ты почему не на уроках?
#38 #344400
Ребзя, подскажите книгу, которая легко описывает матан? Студент 3 курс, понял, что нужно подтянуть знания.
69 Кб, 640x480
#39 #344403
>>344409>>344454
#40 #344404
>>344400
mathprofi, если ты совсем даун
#41 #344409
>>344403
Спасибо.
#42 #344427
Как приучить себя решать матан 8 часов?
>>344434
#43 #344434
>>344427
Для начала понять, что нет никакого "матана" в 8 классе.
>>344436>>344477
40 Кб, 600x600
#44 #344436
>>344434
Че? В каком ещё 8-м классе?
#45 #344454
>>344403
дерьмо
#46 #344477
>>344434
*в восьмом часе.
#47 #344481
>>344166
лол вот это реал дил труъ, и как всегда американец, а рашкованы только по двачам тупят, даже и близко никого нет!
>>344498
11 Кб, 260x306
#48 #344482
#49 #344498
>>344481

>американец


Тем временем либерашка продолжала жидко конфузиться. Впрочем, ничего нового.
>>344502
#50 #344502
>>344498
Ну хуй знает. Яу тоже вот как в девяностые осел в некоммунистической СШАшке, утёкши из не менее некоммунистического Гонконга, так и не собирается оттуда съезжать, судя по всему. И он такой не один. Мурикане оне сука многонациональные. Почему-то пакинстанцы и иранцы тоже не не родине реализуются, как и рашкованы зачастую. В общем под изучение ситуации нужен грант. А без гранта тут нехуй даже обсуждать.
>>344516
#51 #344516
>>344502
Так он же в австралии рос и математическим соком наливался. Можно в принципе заключить, что он и там реализовался, если бы в сшаку не переехал. Но хуй знает, все равно в австралии, что и в рашке, никто не реализовывается. Только в сшаке почему-то. Даже не в гейропе.
1 Кб, 198x60
#52 #344552
Суп, сай. Помогите школьнику вывести сумму ряда. Ну или дайте хотя бы наводок, как это вообще происходит.
156 Кб, 1024x600
#53 #344554
Тут очепятка про а куб или нет?

>>344552
Берешь н=2 и тупо считаешь ряд. Потом берешь н=3,4,5,6....
#54 #344557
>>344552
Что значит "вывести сумму ряда"? Ты задание напиши нормально, от себя хуйни не выдумывай.
>>344579
#55 #344579
>>344557
Доказать тождество пикрилейтед.
>>344590>>344669
765 Кб, 768x432
#56 #344581
Поясните. Фазовое пространство https://en.wikipedia.org/wiki/Phase_space и пространство состояний https://en.wikipedia.org/wiki/State-space_representation в чем разница? И там и там динамическая система представляется в виде своих координат.
>>344584
#57 #344584
>>344581
В первом координаты и импульсы, во втором координаты и скорости
>>344586
35 Кб, 450x285
#58 #344586
>>344584
Хм. Все равно разница как-то неочевидна. Вот пикрелейтед - это фазовое пространство или пространство состояний и почему?
>>344591
#59 #344590
>>344579
Методом мат. индукции пользуйся.
>>344602
#60 #344591
>>344586
То чем ты мучаешься называется общим словом динамические системы. И да, по второй ссылке я не так понял, думал, что под этим подразумевается конфигурационное пространство, а там какая то хуета из теории управлений.
>>344594
#61 #344594
>>344591
Т.е. ты хочешь сказать, что это одно и то же?
>>344597
#62 #344597
>>344594
Нет, я хочу сказать, что лучше оперировать понятием динамическая система, в том числе потому что она хорошо определена.
>>344599
#63 #344599
>>344597
Ок. А как тогда называть пространство координат динамической системы?
>>344603
#64 #344602
>>344590
Нужно найти это имея на руках только сумму ряда.
>>344675
#65 #344603
>>344599
Именем своей мамаши называй. Еба, не существует такого понятия, просто такого нет забудь. Это же картофан ебаный, ты чё педор ашка не понимаишь??
>>344607
116 Кб, 1077x643
#66 #344607
>>344603

>не существует такого понятия, просто такого нет забудь


Да неужели. Динамические системы, описываемые дифурами, обычно представляются в виде своего фазового пространства, тот же учебник Арнольда на этом построен. Далее, именно таким образом можно представить модели лотки-вольтерры и т.п. околоэкологическую хуйню. Далее, читаю я пикрелейтед и что вижу - большинство моделей экологических процессов построено на пространстве состояний из теории управления. В т.ч. та же модель лотки-вольтерры. Я и хочу понять, в чем разница, есть она вообще или это одно и то же? В соотв. статьях в педивикии об этом вообще нет ничего, в других местах так же не нашел. Тут возможны 2 варианта - либо "ЭТО БЛЯДЬ КЛАССИКА, ЭТО ЗНАТЬ НАДА!!", либо (что нереально) этот вопрос до меня никем не ставился в принципе.
>>344613
#67 #344610
Боже мой, говорите просто пространство и всё. Состояния параметризуются точками пространства.
#68 #344611
>>344552
Возьми формулу для $$\sum_{k=1}^{n} x^{-k}$$ геометрическая прогрессия и вычисли производную, это будет формула для суммы из производных $$\sum_{k=1}^{n} -kx^{-k-1}$$. Домножь на $$-x^{n+1}$$ - получишь сумму для $$\sum_{k=1}^{n} kx^{n-k}$$. Подставив, например, x=2 обнаружишь своё равенство. Вообще, для того чтобы творить такую грязь гугли производящие функции, норм такая тема дискретки, полезная для современной математики в том числе.
#69 #344613
>>344607

>дифурами


Картофан, нинужно, затрален.
>>344614
#70 #344614
>>344613

> Картофан


Вербитка, ты что ли?
27 Кб, 880x715
#71 #344619
Таки вопрос остается открытым:
1) Есть динамическая система.
2) В чем разница ее представления в виде фазового пространства и в виде пространства состояний?
Тралей, школуйню и прочее хуепутоло сразу нахуй, интересен ответ по-существу.
#72 #344622
>>344611
Что за нотация?
>>344623
#73 #344623
>>344622
TEX мазерфакер ду ю спик ит?
>>344633
#74 #344633
>>344623
Пока нет.
#75 #344640
>>344611
Ебать ты картофан))0
>>344647>>344735
#76 #344647
>>344640
Вербитоблядь закукарекала заместо авроры))
120 Кб, 642x711
96 Кб, 1126x386
1412 Кб, Webm
#77 #344648
Короче, чем больше гуглю, тем больше запутался. В одном месте (пик 1) явно гоаорится, что фазовое пространство и пространство состояний это одно и то же, в другом (пик 2) - что это разные вещи. В чем жи правда?
>>344655
#78 #344651
>>344611
Если ты ещё тут, объясни, пожалуйста, каждый шаг. Почему ты делал именно так, а не иначе?
>>344654>>344735
#79 #344654
>>344651
Блять, иди в /un лучше, там тебе распишут решение. Задачка твоя тривиальная, решатся методом мат. индукции. Загугли метод математической индукции, посмотри примеры решения подобных задач. Ну ты даун что ли совсем?
>>344663>>344667
#80 #344655
>>344648
В сигсах, братишка, в сигмах.
#81 #344663
>>344654
Очвидно, решение этой задачи по индукции не имеет никакой ценности, потому что если бы ответ не был заранее посчитан, ты бы до него никогда не догадался.
>>344665>>344669
#82 #344665
>>344663
Даун это ты. Перечитай посты по теме.
>>344667
#83 #344667
#84 #344669
>>344663

>если бы ответ не был заранее посчитан


Но ответ просчитан заранее, дебил. Школопердяй тут высрал >>344579 , что нужно доказать тождество. И всё.
>>344675>>344684
#85 #344675
>>344669 => >>344602
Просто иди нахуй, зеленый.
#86 #344684
>>344669
Бля ты правда тупой или притворяешься? Есть ряд sum kn^(n-k), надо найти closed form для этого выражения.
#87 #344735
>>344651
В смысле, понятно вообще что сделано? Или ты спрашиваешь почему именно так? Ну это суть всей этой техники, взять известную сумму/ряд (в данном случае конечную геом.прогрессию) и интегрированием/дифференцированием/умножением/сложением получить требуемую сумму/ряд. Тогда выражение суммы/ряда, если оно есть, получается соответствующей последовательностью действий с выражением для исходной суммы/ряда, которая известна.
>>344640
Но..там же теневое исчисление с такими штуками бывает, алгебра хопфа даже на полиномах есть!
>>344743>>344813
#88 #344743
>>344735

>Но..там же теневое исчисление с такими штуками бывает, алгебра хопфа даже на полиномах есть!


Это вообще о чем? "Алгебра Хопфа на полиномах" это я понимаю афинная групповая схема, а все остальное про что?
>>344746>>344758
#89 #344746
>>344743
Ладно, я уже сам нашел. Вопрос меняется на зачем оно нужно и где про него написано.
>>344758
#90 #344758
>>344743
>>344746
Ну, я вообще прочитал это в книге Ландо "Введение в дискретную математику". Ну, применяется в комбинаторных равенствах всяких, есть связи с разными замечательными последовательностями полиномов, и теориями операторов на них.
#91 #344790
>>344552
Можно без производных и производящих функций, а также мат. индукций на отъебись.
Попробуй представить выражение под суммой как сумму(умножение на целое - это сумма). И нарисовать сию двойную сумму по слагаемым в виде матрицы - по строкам будут внутренние суммы(выйдет-таки треугольник). Далее выполни суммирование сначала по строкам(треугольник схлопнется в одну строчку)Увидишь геометрическую прогрессию ты, а потом суммируй эту строчку. Надеюсь, я ясно разьяснил. Сии операции можно выполнить/записать путем перестановочек знак сумм и выкрутасов с индексами, но это довольно сложно осознать неискушенному в матанах.
Такой вот алгебраический хинт ушами.
inbefore картофан
#92 #344812
>>344611
Допустим. А что дальше-то с этим делать? Не понимаю.
>>344813
#93 #344813
>>344735
>>344812
Не заметил твоего ответа. Всё понял, спасибо.
#94 #344822
>>343958 (OP)
Почему эллиптические кривые так важны и пропиарены, а кривые уравнений высших степеней нет? Лишь потому что уравнения 3 степени легко решать?
#95 #344827
>>344822
Потому что они сыграли ключевую роль в доказательстве теоремы Ферма.
https://en.wikipedia.org/wiki/Fermat's_Last_Theorem#Connection_with_elliptic_curves
#96 #344851
Пасаны, есть какой-нибудь онлайн сервис, чтоб интегралы символьно считать? Желательно с туториалом. Помню, что в мапле можно было, но ставить его запало что-то.
Надо срочно проверить один картофельный интеграл, я хоть и справился, но чёта думаю, что мог обосраться жиденько.
#97 #344853
Чем отличается функционал от функции?
>>344854
#98 #344854
>>344853
ничем, просто функционалом чаще называют отображение из какого-то функционального пространства на действительную прямую
>>344857
#99 #344857
>>344854

> в поле/кольцо, над которым это функциональное прстранство рассматривается


фикс
>>344858
#100 #344858
>>344857
ну окей, тащемта если ты меня поправил, тогда поясни мне вот что, а то я хуй простой
Какие ещё есть поля кроме действительных и комплексных чисел? Часто вижу, что теоремы доказываются, скажем, для произвольного поля, но примеров полей, кроме вышеперечисленных, не дают.
>>344859>>344860
#101 #344859
>>344858
На ум всплывает только неопределённое кольцо N.
>>344876
#102 #344860
>>344858
Поле - это любой объект, удовлетворяющий аксиомам. Q, С и R лишь частные примеры. Если хочется "не очень традиционных" полей, можешь рассмотреть поле вычетов по модулю p Zp или любое расширение С, R, чего угодно ещё.
#103 #344876
>>344822
Придумай применение для кривых восьмой степени.

>>344859
Репорт петушка
>>344922
#104 #344880
>>343958 (OP)
Вся математика опирается на три фундаментальных значения: ноль, единица и бесконечность.
>>344895
#105 #344895
>>344880
Толсто.
>>345124
#106 #344898
>>344894
Иди нахуй.
#107 #344922
>>344876

>Придумай применение


Так если реальное применение находят только какие-то частные примеры кривых или поверхностей, то чем тогда оправдывается пафос алгема как "центральной науки"?
>>344924>>344930
#108 #344924
>>344922

>то чем тогда оправдывается пафос алгема как "центральной науки"?


Твоей тупостью и тупостью тебе подобных, которые считают, что в математике есть что-то центральное.
>>344943
#109 #344930
>>344922

> чем тогда оправдывается пафос алгема как "центральной науки"?


Какой пафос? Почему центральной? Что ты несёшь вообще?
>>344943
#110 #344932
Поясните, почему формально полученные ряды/произведения/разложения нужно еще проверять на сходимость, причём они могут не сходиться вообще - в чём не состыковочка? В чём "упущение" акробатики, которая позволяет получать подобные выражения?
>>344944>>344945
#111 #344943
>>344924
>>344930
Детям - мимо.
#112 #344944
>>344932
В алгеме, кстати.
#113 #344945
>>344932
Формальные ряды не учитывают сходимость.
>>344953
#114 #344953
>>344945
Но мы же их получили эквивалентными преобразованиями, как при преобразовании обычных выражений. Когда мы оперируем подобным образом над алгебраическими выражениями дополнительно проверять не надо - выражения тождественны. А тут у нас вроде бы тождество, а при проверке оно может оказаться ложным.
В чём заложена скрытая разница? Может мы упускаем/подразумеваем какие-то детали предельных переходов?
>>344971
#115 #344971
>>344953

>Но мы же их получили эквивалентными преобразованиями, как при преобразовании обычных выражений.


В них тоже не учитывалась сходимость.

>Когда мы оперируем подобным образом над алгебраическими выражениями дополнительно проверять не надо - выражения тождественны.


Спорно. Можно например переопределить отношение '='. То есть снова необходимо учитывать модель. В простых случаях она используется по умолчанию.

>А тут у нас вроде бы тождество,


Формальное.

>а при проверке оно может оказаться ложным.


Более того при различных видах топологий (fine-coarse) - сходимость может быть, а может не быть. Но опять: формальные ряды – игнорируют сходимость.
#116 #345005
"Эквивалентные преобразования" конечно не сохраняют сходимость, в этом фишка. Тем не менее, вопрос несколько глубже на самом деле, потому что это действительно несколько странный эффект, когда равенство чисто алгебраических выражений как функций от x зависит от этого самого значения x. Например, $\frac{1}{1-q}$ и $1+q+q^2+q^3+\dots$ равны как обычные конечные значения при $|q| < 1$, ещё можно принять их равенство как бесконечностей при $q=1$, но почему первое выражение обычное число при всех остальных $q$, а ряд расходится к бесконечности?
#117 #345041
>>344822
Не слушай уебков тебе отвечавших, они ничего не знают. Эллиптические кривые важны и пропиарены в основном из-за того, что это абелево многообразие, т.е. на них есть естественная структура группы (если выбрать точку, которая будет нулем). С комплексной точки зрения просто тор. По сути это самый простой из нетривиальных примеров многообразий, который еще можно совершенно в-наглую пощупать и очень много узнать.
Но в алгеме и комплексной геометрии, конечно, еще куча куча многообразий не менее пропиаренных. А вне так много шума из-за разных применений в криптографии и теории чисел (которые обычно следуют из группового закона).
>>345068
#118 #345056
Почему все математики очень тупые?
>>345071
#119 #345068
>>345041
Спс, аллах тебя не забудет.
#120 #345071
>>345056
Да потому что у них очень абстрактное мышление, оторванное не то, чтобы от трёхмерного пространства, но и, в некоторых конкретных случаях, от реальности вообще. Та же самая новая хронология Фоменко, теория струн, а в более запущенных случаях (вроде моего) - математическое устройство мира(т.е. все, что мы видим в мире - это сложные уравнения, возможно заложенные кем-то изначально, но не все уравнения в природе нам дано понять/узнать)
>>345076
#121 #345076
>>345063
Но вот же - >>345071 - живой пример!
>>345078
#122 #345078
>>345076
Ну, каждый ебанут в свою сторону. Кто как может.
>>345930
#123 #345083
Что делать, если я придумал фигню? Публиковаться?
>>345084>>345086
sage #124 #345084
>>345083
Смотря что за фигня, и как придумал.
#125 #345086
>>345083
Можно сказать с большой степенью уверенности, что то, что ты придумал уже отражено в том или ином виде в научных публикациях или математической литературе. Так что не спеши радоваться.
>>345091
#126 #345091
>>345086
А если я действительно придумал что-то новое, мне денег дадут?
>>345092
#127 #345092
>>345091
Нет, скорее всего.
>>345093
#128 #345093
>>345092
А на что тогда живут математики? Глупые.
>>345095
#129 #345095
>>345093
Так математики, а не "математики". Те кто кончил вузик, пошёл на постгредуейт/аспирантуру, и далее по накатанной. Кто встроен в систему, короч.
>>345098
#130 #345098
>>345095
То есть математик - это хуйня
>>345099
#131 #345099
>>345098
Ну да, тянки то ним не текут, увы.
>>345101
#132 #345101
>>345099
1) сказал какую-то математическую поеботу
2) ????
3) PROFIT
>>345107>>345113
#133 #345107
>>345101
1) сходил в офис
2) сделал никому не нужную хуйню
3) пришёл домой
4) ???
5) ЗАРПЛАТА
6) повторяешь 1) - 5) в течении 50 лет
7) умираишь
#134 #345113
>>345101
Вряд ли удастся впечатлить тню какой-нибудь математической поеботой, если ты об этом, но если и не об этом, то всё равно напишу. Для большинства из них математика - это такой себе вид задротства, вроде мморпг или сборки кубика-рубика 11✕11, только с цифрами и фигурками. Ты произведёшь впечатление чудака: "какой ненормальный, зачем он рассказывает про свои гомологии, странный" - подумает тня.
Другое дело физика, биология или психология, - в общем те дисциплины, которые находятся на острие практической науки и активно популяризируются. Если ещё и речь хорошо поставлена и тема актуальна, то успех тебе гарантирован.
70 Кб, 451x604
#135 #345120
>>345113

>Другое дело физика, биология или психология


Какие отчаянные фантазии. Да тот же самый результат, если эта "наука" не имеет отношения к модному айфончико-ориентированному околоайти, бинарным опционам или прочей чепухе, которая неразрывно связана с доходом от 50к$ в месяц.
>>345128>>345141
#136 #345124
>>344895
Толсто у тебя за щекой
#137 #345128
>>345120

>имеет отношения к модному айфончико-ориентированному околоайти, бинарным опционам


Это по большому счету относится к совсем уж ванильным тпшкам. Я скорее имел ввиду обычных няшек-стесняшек из университетов средней руки.
#138 #345136
В математике есть раздел "дискретная математика" или этот винигрет придумали пидарасы для технариков?
>>345138>>345144
#139 #345138
>>345136
Есть. Это вся математика, которая не касается понятий бесконечности и непрерывности.
>>345193
120 Кб, 873x655
#140 #345141
>>345113
>>345120
Вы совершенно ничего не понимаете в тянках. Да и о произведении впечатления вообще. Совершнно не важно чем ты занимаешься и о чем говоришь. Это с равной вероятностью может быть воспринято как унылое задротство и как охуенная.
Кстати, вбрасываю вполне себе солидного математика, выходца из России.
#141 #345142
>>345141
Это с равной вероятностью может быть воспринято как унылое задротство и как охуенная интеллектуальность в зависимости от харизмы и предпочтений собеседника.
#142 #345144
>>345136
Нет, разумеется. Такого раздела нет. Математические объекты не делятся по своей природе на "дискретные" и "непрерывные". Есть совершенно неожиданные примеры такого рода недвойственности. Например, в науке Громова группы можно представить в виде графа Кэли и изучать с помощью... метрической топологии.
>>345152>>345158
#143 #345148
>>345141
Ты намекаешь на то, что тням похуй, что им говорят, главное слышать плавно текущую речь и видеть смазливое ебальце перед собой? Так это вроде понятно. А что если ты не является обладателем ни того ни другого?
#144 #345152
>>345144

>Математические объекты не делятся по своей природе на "дискретные" и "непрерывные".


Ты так говоришь будто есть математические боги, которые запрещают делить объекты на "дискретные" и "непрерывные". Если смотреть с позиции, - что если что-то из одной области можно каким-то образом представить в другой области, то тогда области нельзя отличить; выйдет так, что из математики и вовсе нельзя выделить ни одного раздела.
#145 #345157
>>345141
Маттрахер.
#146 #345158
>>345144
А множества не делятся на счётные и континуальные.
#147 #345160
Не хочу преподавать математику студентам-долбоёбам, хочу заниматься только исследованиями (какими смогу).
В России это возможно? Доход неважен, лишь бы после получения диплома хватало на еду и снимать жильё.
При этом меня не интересует статистика, теория вероятностей и прочая поебень - в экономическую контору не пойду.

Сейчас второй курс столичного ВУЗ-а средней руки. Преподы ретрограды сумасшедшие, могу спросить только у вас.
>>345162>>345163
#148 #345162
>>345160
А недолбоёбам готов преподавать? Можно ещё умудриться работать не преподавая где-то в стекловке, в смысле не обязательно каждый год браться вести курс лекций семестровый, но какая-то педагогическая практика всё равно будет когда ты будешь учиться в аспирантуре, например.
#149 #345163
>>345160

>хачу заниматься исследованиями, пока низнаю какими и нахуя они


>это, это и это сразу нахуй


>вы токо оплатите мне хату и ништяки


>а я пока двач поскролю

#151 #345173
>>343964
Соус автора
#152 #345175
Хорошо знаю три иностранных языка, могу со средними усилиями полностью выучить и запомнить почти тысячу иностранных слов за день. Это значит, я смогу стать и хорошим математиком?
>>345181
#153 #345181
>>345175
нет
>>345182
#154 #345182
>>345181
Математику разве нужно что-нибудь, кроме памяти?
#155 #345184
>>345182
Картофель и водочка
>>345202
#156 #345186
>>345182
А что ты запоминать собрался: формулы, теоремы, значки? Без понимания, как всё получается и для чего служит, тебе это не нужно. Сгенерировать что-либо новое ты будешь неспособен. А воспроизведение материала по памяти по большому счету нахуй никому не нужно. Всегда можно открыть учебник и посмотреть, что требуется.
#157 #345193
>>345138
Убери бесконечность.
#158 #345202
>>345184
Картофан и водовка
>>345214
#159 #345203
>>345182
Воображение и культурная база
234 Кб, 628x1298
Помогите понять задачу, пожалуйста #160 #345213

>1.Сформируйте систему линейных уравнений (то есть задайте матрицу коэффициентов A и свободный вектор b) для многочлена первой степени, который должен совпадать с функцией f в точках 1 и 15. Решите данную систему с помощью функции scipy.linalg.solve. Нарисуйте функцию f и полученный многочлен. Хорошо ли он приближает исходную функцию?


Я решил, получилось 3.43914511, -0.18692825

Второе часть не могу понять

>2.Повторите те же шаги для многочлена второй степени, который совпадает с функцией f в точках 1, 8 и 15. Улучшилось ли качество аппроксимации?


Где взять многочлен второй степени?
>>345454>>345513
#161 #345214
>>345202
Водофан и картовка.
#162 #345384
>>345168
ромыч завис на постмодерне, а ведь уже не 70-е годы.
>>345423
#163 #345397
Парни, какие есть годные видео-лекции на русском (по любым направлениям и разделам математики)? НМУ-шные? Там по десятку лекторов на предмет, какие выбирать?
>>345451>>346078
#164 #345423
>>345384
В натуре. Постоянно ссылается и цитирует, а говорил, что против постоянного цитирования.
https://youtu.be/fJJ4W-0ATn8?t=1352
22:32
#165 #345430
>>345168
Ну поскольку я потратил время на прочтение, получите рецензию.
Не зная Вашей даты рождения и соответственно Вашего гороскопа, а как известно любому астрологу, человек обычно рассказывает только свой гороскоп ;))
Отмечу следующее. Книга написана достаточно живо и красочно, что намекает на хорошие аспекты венеры с меркурием. При этом есть концепция - узор(фрактальность, склейка), что вероятно говорит о пребывании планет в знаке стрельца.
Перечисление - эпилепсия, паранойя, шизофрения, говорит о выделенности тем для автора.
Что такое эпилепсия, как неосознанные выплески энергии. В отличие от маниакальности, когда энергия направлена на какую-то цель. То есть проблемы с Марсом и/или Плутоном. Ну и собственно весь текст, скорее описательный, что говорит о преобладании нейтральных и женских энергий. Кстати аватарка с преобладанием оранжевого, говорит об активности 2 чакры и энергиях Меркурия и Венеры. Короче основная точка сборки Вашей личности - там.
Параноидальность - о желании понять структуру мира, это выделенность Сатурна. А шизофрения как раскол - о слабой проработке и осознанности энергий сверхличных планет Урана и Плутона.
#166 #345444
У математиков волосы стали дыбом! Ваш IQ выше 1448, если вы решите эту задачу без гугла: три рыбака легли спать, не поделив улова. Проснувшийся ночью первый рыбак решил уйти, взяв свою долю. Но число рыб не делилось на три. Тогда он выбросил одну рыбу и забрал себе треть. Второй и третий рыбаки поступили аналогичным образом. Вопрос: какое наименьшее число удовлетворяет исходному количеству выловленной рыбы?
#167 #345447
-Откуда взялись струны или браны если они есть
-Почему планковская длина считается абсолютным нулем
-Зачем нужны тахионы если есть суперсимметрия
-Почему гравитация не квантуется в если есть гравитоны
-Почему космические тела вращаются вокруг собственной оси
-Почему электрон не падает на ядро теряя энергию
>>345450
#168 #345450
>>345447
Это лишь разные виды узоров. Зачем нужны узоры, каждый сам видит для себя. Физика она как индия, там все есть, каждый берет что хочет.

>>345444
Одна рыба.
>>345453
#169 #345451
>>345397
Выбирай НЕ прасолова.
>>345456
#170 #345453
>>345450

>одна рыба


Нет.
>>345461
#171 #345454
Бамп >>345213
#172 #345456
>>345451
А кого?
#173 #345461
>>345453
Обоснуй. 1 не делится на 3. Он выбросил рыбу и забрал себе треть нихуя.
>>345463>>345466
#174 #345463
>>345461
Остальные не смогут сделать также.
#175 #345464
>>345444
Минимальный улов естественно достигается, если каждый рыбак забрал минимально возможное число рыбешек, а именно одну. Мне лень считать.
>>345466
#176 #345465
Вообще не ясно что значит "Второй и третий рыбаки поступили аналогичным образом". Если подразумевается что последний рыбак откинул одну рыбу и взял треть, то это максимум тупое поведение.
>>345466>>345467
#177 #345466
>>345465
Нет.
>>345464
Нет.
>>345461
Нет.
>>345468
#178 #345467
>>345465
Рыбак же не знает что все ушли, он думает что это неподеленый улов на троих.
>>345472
#179 #345468
>>345466
Пидора ответ
>>345466
Пидора ответ
>>345466
Пидора ответ
#180 #345470
>>345444
40
40 = (((1x3 + 1)x3 + 1)x3 + 1)
>>345476
#181 #345472
>>345467

> неподеленый улов


Который за одну ночь уменьшился в три раза? Рыбак после сна забывает все что произошло в прошлый день?
>>345476
#182 #345473
>>345444
Аноны, что-то еще про про продавца шапок есть! Типа фальшивую купюру ему дали и нужно узнать насколько он попал. Или наоборот, покупатель попал. Кто помнит? Охуетельно сложная задача, 100 ответов и все неверные. Кажется просто, а ведь нескольких лет мехмата уровень!
>>345475
#183 #345475
>>345473
Дали продавцу шапок сто фальшивых купюр, а они все как раз.
>>345477
#184 #345476
>>345470
Нет.
>>345472
Нет.
>>345478
#185 #345477
>>345475
Это загадка или отгадка?
>>345479
#186 #345478
>>345476
Что нет? Улов не уменьшился? Или рыбак забывает? Улов не может не уменьшится по условию задачи. Про склероз у рубака ничего не написано.
>>345480
#187 #345479
>>345477
У мужика кепка-пятиклинка стоит 200 рублей, покупатель хочет купить её, но у него только 500. Подзывает мужик мишу вербитского, выпидоренного за космополитизм из вышки и говорит: "Беги, еврейка, скорее к тетё маше, разменяй". Побежал миша со всей прыти, всё разменял на сотки, прибежал обратно. Отдал мужик сдачу и кепку, сидит, отдыхает. Через полчаса заходит тётя маша и говорит, что купюра фальшивая, мужику пришлось заплатить всё из своего кармана. Был конец рабочего дня, мужик наказал вербитскому выяснить в чём дело и определить до завтра где же его наебали, а сам домой пошёл. Помоги мишке найти сумму наёба.
#188 #345480
>>345478
Всё нет.
>>345483
#189 #345481
>>345479

>мужику пришлось заплатить


Вернуть т.е. тётке все 500 рублей.
#190 #345482
>>345479

> сумму наёба


Проеб - пятсот фальшивых рублей. Если сможет их кому-то впарить как пятисотку, то не проебал. Если за половину - проебал 250. Не сможет - 500.
#191 #345483
>>345486
#192 #345484
>>345444
Тут получается диофантово уравнение
8x - 27y = 38. НОД(8,27) = 1, следовательно оно неразрешимо в целых числах.

>Вопрос: какое наименьшее число удовлетворяет исходному количеству выловленной рыбы?


Ответ: никакое
>>345486>>345572
#193 #345486
>>345484
Нет, дурак.
>>345483
Нет.
>>345488
#194 #345488
>>345486
Нахуй пошел.
>>345494
#195 #345489
>>345494
#196 #345494
>>345489
Ты дурак?
>>345488
Нет.
#197 #345495
>>345479
Покупатель наварил 300 рублей плюс шапку. Значит продавец очевидно попал на 300 плюс смотря как считать стоимость шапки.
>>345496>>345497
#198 #345496
>>345495
Стоимость шапки - цена шапки, так как продавец проебал возможность ее продать за нормальные деньги.
>>345497>>345498
#199 #345497
>>345495
Нет.
>>345496
Нет.
>>345499>>345500
#200 #345498
>>345496
Ну тогда 300 + 200 = пицот
#201 #345499
>>345497

> Нет.


Пидора ответ.
>>345501
#202 #345500
>>345497
Что именно нет?

Соседка осталась при своих, да или нет?

Покупатель наварил 300 руб плюс шапку, да или нет?

Продавец отдал соседке своих 300 плюс потерял шапку, которую мог бы продать за 200, да или нет?

Обоснуй, а то пока мне кажется что это просто пидора ответ.
>>345503
#203 #345501
>>345499
Нет.
#204 #345503
>>345500
Всё нет.
>>345505
#205 #345505
>>345503
Блин ! Ну , что Вы все соседские деньги , соседские деньги ....!
Вместе с остатком , вместе с остатком ..... ! Какие они к чертям соседские ? Какой к чёрту остаток ?
Ребята это деньги ПОКУПАТЕЛЯ !!!! Реальные или фальшивые выяснится позже . Соседка лично своего ни чего не давала ! Её роль заключается всего лишь в размене денег. И продавец всего лишь вернул 15 р. его же денег ( покупателя ) , только уже реальных . А продавец заработал уже свои личные , кровные 10 р. с продажи шапки . В результате обнаружения фальшивки Продавец отдаёт свои кровно заработанные 10 р. и добавляет свои же 15 р. и отдаёт соседке ( 25 р. ). Вы спросите почему соседке ? Потому что она их получила в данном случае от продавца , а не от покупателя . Кроме того продаётся одна шапка , не 10 и не 1000 и не .....
А именно одна из условия задачи и цена ей 10 р . В результате афёры продавец имеет фальшивую купюру в 25.р которая ни кому не нужна !!!! Продавец остался и БЕЗ ШАПКИ И ЕЩЁ ОТДАЛ СВОИ 25 р.
Ущерб продавцу составляет 35 р.
>>345506>>345705
#206 #345506
>>345505
Нет.
>>345507
#207 #345507
>>345506
Вы зациклились на элементарных действиях . Получил , отдал . Получил , отдал !!! А ведь я вам изначально говорил обратите внимание на условие задачи : Один продавец ( сам по себе ) , ОДНА шапка ( понимаете ОДНА У ЭТОГО продавца ). Стоит конкретные 10 р.

Я ведь разжёвывая Вам не зря приводил варианты . В варианте ЖУЛИК – ПРОДАВЕЦ ( без размена) , продавец теряет 25 р . В варианте ЖУЛИК – ПРОДАВЕЦ ( с разменом в магазине розничной торговли ) , продавец теряет 25 р. Читайте выше мои комментарии .

В третьем варианте ( конкретные условия задачи ) в процессе сделки фальшивые деньги номиналом в 25 р. ЖУЛИКА (ДЕНЬГИ ЖУЛИКА ,А НЕ СОСЕДКИ ) плавно перетекают через продавца к соседке , где они удачно меняются на реальные и опять через продавца возвращаются к Жулику в виде ПРОДАННОЙ шапки и реальных 15 р. ПРОДАВЕЦ ПОЛУЧАЕТ ЧЕСТНО ЗАРАБОТАННЫЕ 10 р. С ПРОДАЖИ ШАПКИ. ХОРОШЕНЬКО ЗАПОМНИТЕ !!!! ЭТА ДЕСЯТКА УЖЕ ЕГО СОБСТВЕННОСТЬ !!! ЭТО ДЕНЬГИ НЕ СОСЕДКИ КОТОРЫЕ ПРИШЛИ ПОСЛЕ РАЗМЕНА , ЭТО ДЕНЬГИ КОТОРЫМИ РАСПЛАТИЛСЯ ЖУЛИК ЗА ШАПКУ .
Поэтому как только к этой десятке Вы прибавляете ещё свои 15 р.( есть у продавца в данный момент ещё свои деньги или их нет РОЛИ НЕ ИГРАЕТ ) и отдаёте ( или 10 р. и будете должны 15 р. ) соседке 25 р. ПРОДАННАЯ ВАМИ ШАПКА ИЗ СТАТУСА ПРОДАННОЙ ПЛАВНО ПЕРЕХОДИТ В СТАТУС ДАРОМ ОТДАННОЙ !!!!! НЕТ ШАПКИ , НА РУКАХ ФАЛЬШИВЫЕ 25 р. Ну и на сколько Вас нагрели ????

Ребята Вы зацикливаетесь на деньгах , ДАЛ ,РАЗМЕНЯЛ ,ОТДАЛ , ВЕРНУЛ – фигурируют конкретные суммы . И в тот же момент Вы теряете из виду саму шапку . ШАПКА – ТОВАР С КОНКРЕТНОЙ ЦЕНОЙ , ВМЕСТО ШАПКИ – ТОВАРА ПРОДАВЕЦ ПОЛУЧАЕТ ДЕНЬГИ – ПРИБЫЛЬ . И ЕСЛИ ОТДАТЬ ДЕНЬГИ – ПРИБЫЛЬ ,ДА ЕЩЁ ДОБАВИТЬ СВОИ , ТО ПРОДАВЕЦ ТОВАР – ШАПКУ ПРОСТО ОТДАЛ ДАРОМ !!!!! ( НЕ ЗАБЫВАЕМ РЕЧЬ ИДЁТ О КОНКРЕТНОЙ СДЕЛКЕ ЖУЛИК – ПРОДАВЕЦ -(РАЗМЕН )- ПРОДАВЕЦ -ЖУЛИК ).

В данном случае ШАПКА досталась ЖУЛИКУ даром , а соседка урвала 25 р .
>>345508>>345705
#208 #345508
#209 #345509
>>345444
10 рыб.
#210 #345511
Сие Я– автор поста номер 345444 (>>345444).
Мой ответ на все ваши предположения: нет. Вы не достигли правильного ответа.
Внимание, правильный ответ: наименьшим числом, удовлетворяющим заданным условиям, является (-2). Рыбаки выловили -2 рыбы или две антирыбы.

Спасибо за внимание, глупцы, моё дело сделано, Я ухожу.
>>345519
#211 #345513
Бамп >>345213
#212 #345519
>>345511
Ты охуел? Чем тебя например не устраивает -29 рыб? А, сука?
#213 #345520
Братья ананасы, как взять первообразную этой функции e^(x^2)?
>>345521
#214 #345521
>>345520
(e^(x^2))/2x^2
>>345526
#215 #345526
>>345521
не выражается в элементарных функциях, а так выражается через функцию ошибок, путем интегрирования по частям.
#216 #345527
>>345479

>свербицкий



а почему мишу тут так не любят? он конечно скандалист и русофоб, но математик же международной пробы. у него индекс цитирования кстати - огого, выше кажется только у арнольда был.

как думаете аноны, он где-то с арнольдом одного уровня или все-таки уступает? за сколько бы он решил сто задачек арнольда?
>>345538>>345627
98 Кб, Webm
#217 #345529
>>343958 (OP)
1 сентября 1997 года, в результате деления на ноль в компьютеризированной управляющей системе крейсера USS Yorktown (CG-48) Военно-морского флота США произошло отключение всех машин в системе, в результате чего двигательная установка корабля прекратила свою работу
#218 #345538
>>345527
Арнольд лох, извините. Полный лох.
>>345542
#219 #345542
>>345538
Почему? Он же боролся с бурбаками. А бурбаки как вам?
>>345545>>345641
#220 #345545
>>345542
А бурбаки няшки. Особенно готично-лысый Гротендик и Вейль со своими гипотезами.
>>345553
#221 #345553
>>345545
Хранцузского школьника спросили, сколько будет 2+3? Он ответа не знал, но ответил как его научила бурбакомафия: 3+2, так как сложение коммутативно.
байка от арнольда
>>345582>>345641
#222 #345571
>>345444
25 рыб
#223 #345572
>>345484
следовательно оно неразрешимо в целых числах.
Скорее: следовательно - оно имеет бесконечно много решений в целых числах при любой правой части.
>>345596
#224 #345582
>>345553
А почему вопрошающий счёл, что ответом на вопрос "сколько будет 2+3" не является "3+2"? Школьник, типа, должен был угадать слово, которое загадал вопрошающий?

Пусть сначала научатся точно формулировать вопросы, и только потом спрашивают.
#225 #345596
>>345572
Да, тут я обосрался. Перебором остатков можно подобрать частное решение (25,6).
#226 #345627
>>345527
Нихуя он не международной пробы, да и какое это имеет значение? Важны ведь только результаты. Арнольд, например, международной пробы. Доказывал проблемы Гильберта, унижал бурбаков и прочее быдло, за что ему посмертный рейхспект. Миша решит максимум 2 задачи из тривиума, для него ведь это зашквар. Есть три категории математиков. Первая это так называемая первая культура(низший сорт, 1 уровень). У них принято считать, что самое главное - это новая абстрактная теория. Такая себе чистая математика. На уровень выше стоят прикладники и формалисты, которые пытаются получить результаты в разных областях теми методами, которые есть, и разрабатывают новые, только если это необходимо. Ну и третья категория самая немногочисленная, это такие себе полубоги по типу Перельмана, которые доказывают проблемы специальными детализированными методами.
>>345639>>345640
5 Кб, 279x297
6 Кб, 366x422
#227 #345628
В /уне такие же школьнике как и я, хз объяснят ли. Обоссыте, но помогите..
Нихуя не понимаю смысл производной. Вот например дефолтная ЕГЭшная задача. Правильно ли я думаю?
1. Производительность рабочих есть некоторая функция, а значит у это функции есть график.
2. Нужно найти максимум на это графике. Если, например, мое время не ограниченно, то я могу просто подставлять все значения в функцию и рано или поздно я получу этот максимум.
3. Очень долго производить такие вычисления. Поэтому вместо самой функции можно рассмотреть её аппроксимацию или проще говоря спрямление этой функции.
4. Получив вместо кривой функции обычную прямую или что-то похожее намного проще понять где у неё максимум.
Для этого и нужна производная?
Почему тогда, это спрямление пик 2 вообще не похоже на оригинал пик 1?
https://www.youtube.com/watch?v=Erzf5ktLkMw&index=3&list=PLowFPHQVq5qPL_Nmx5sITxbzoou_k-gO8
#228 #345629
Вы что тут устроили со своими детскими задачками? Еще теорию графов на их решение примените. Долбоебы...
#229 #345630
>>345628
ALLO, DAUN, ETO TI?
Смысл производной в том, что она показывает скорость изменения функции в точке.
#230 #345631
>>345630
Я слышал это уже 500, яснее не стало.
>>345634>>345636
#231 #345634
>>345631
Значит ты дурак.
Открываешь тетрадку тгда и считаешь производную из функции х², может поймешь.
#232 #345636
>>345631
Хотя ладно, поясню дураку почему он дура. Жди пять минут пока я набираю
#233 #345637
>>345628
вычисляешь s', приравниваешь s'=0, находишь иксы и подставляешь их в s.
>>345657
#234 #345638
>>345630

>Смысл производной в том, что она показывает скорость изменения функции в точке


Блядь, где строгое математическое определение, гильберт тебя за ногу?
>>345642
#235 #345639
>>345627

>Нихуя он не международной пробы


Давеча он представлял Россию на International Congress of Mathematicians.
230 Кб, 640x425
#236 #345640
#237 #345641
>>345542
>>345553
Если что, Арнольд в какой-то момент совсем ебнулся и стал нести хуйню на уровне Пахома. Например, про космические лучи, которые мешают компьютерам работать, из-за чего он с ручкой и бумажкой считает гораздо быстрее и "правильнее", чем всякие комплюктеры. Или вот про 3+2.
#238 #345642
>>345638
Пусть есть вещественная функция f.
Пусть есть отрезок [a;b].
Изменением функции f на этом отрезке называется число f(b) - f(a).
Длиной отрезка называется число h = b-a.
Средней скоростью изменения функции называется изменение функции, делённое на длину отрезка.
Символьно, fсредняя ba = (f(b) - f(a)) / (b-a).

Средняя скорость определена для отрезка. Но что если нас интересует скорость в точке? Нужно использовать предельный переход.

Скоростью функции в точке a называется fсредняя ba, когда b стремится к a. Или, эквивалентно, когда расстояние h между b и a стремится к нулю.

То есть fв точке a = limh→0(f(a+h) - f(a))/h
>>345646>>345657
#239 #345643
>>345641
Тебя бы грузовик переехал - ты бы ещё не так ебанулся.
56 Кб, 1024x600
#240 #345644
У тебя есть функция х². При значении икса 1 функция равна 1. Но вдруг тебе захотелось узнать насколько сильно измениться значение функции если к одному добавить одну десятую, относительно этого самого увеличения. Тобишь, насколько сильно увеличится функция относительно её увеличения.
Ты делаешь следующие рассчеты (1+0.1)²-1²/0.1
Посчитаем в лоб. 1.1²=1.21, 1²=1
1.21-1=0.21/0.1=2.1
Вроде все просто.
А если тебе надо узнать насколько измениться значение функции если к единице добавить одну миллиардную, или даже какую то охуллиардную, что тогда? Теперь уже кучу ноликов писать сильно долго, неудобно, да и ошибиться можно. Заменим теперь цифры буквами.
Получится (х+у)²-х/у²=х²+2ху+у²-х²/у=2ху+у²/у=2х+у
Если теперь подставить вместо букв цифры, то можно увидеть, что прибавление охуллиардной части не сильно влияет на результат, а именно 2+почти ноль.
Что значат эти цифры?Это и есть скорость и величина изменения функции по направлению оси икс.
>>345657
8 Кб, 200x301
#241 #345645
Тута про Арнольда заговорили, как я вижу. Вы же про пикрелейтед Арнольда говорите? Читаю я пикрелейтед книжку, и что-то понимаю что нихуя не понимаю кариночка-сложно.webm. С первой страницы диффеоморфизмы, фазовые потоки, вот это все. Что можете посоветовать по дифурам попроще?
>>345769
#242 #345646
>>345642
Это не строгое
>>345647>>345651
#243 #345647
>>345646
Нахуй строгость. Он сперва понять должен что от него хотят, а потом уже строгость
#244 #345648
>>345628

> можно рассмотреть её аппроксимацию


Апроксимируют когда функция не известна, а есть только несколько ее значений. У тебя же функция известна, следовательно никаких апроксимаций не нужно.
Твои графики это ни какие не "спрямления". Первый рисунок это график целевой функции, на которой нужно найти максимум. Только у тебя там как-то оси перепутаны, более показателней было бы х по горизонтали пустить. Второй график это график производной функции. На нем интрестно нулевое значение, которое соответсвует экстремуму функции.
>>345657
#245 #345649
Меня всё мучает один вопрос: нахуй в современном мире нужна ебучая евклидова геометрия и её методы?
>>345652>>345653
#246 #345650
>>345641
Лол блядь
#247 #345651
>>345646
Это строгое.
>>345668
#248 #345652
>>345649
Сгори в адском пламени.
>>345659
#249 #345653
>>345649
поясни в убедительных тезисах, почему она ненужна?
>>345659
#250 #345657
>>345644
>>345648
>>345637
>>345642
Спасибо, но у меня еще тупее вопрос. Как с помощью понимания как возрастает функция я могу найти её максимум?
>>345665>>345671
#251 #345659
>>345653
Сами её методы - хуйня, переходящая во что-то осмысленное только в алгебраической форме. Зачем людям знать о таком недоразумении, как Евклид и о его работах, когда у нас есть линейная алгебра?

>>345652
А вот и мусор прибежал.
>>345661>>345666
#252 #345661
>>345659
Двачую
#253 #345665
>>345657
Максимум/минимум функции это значение функции при котором соседние участки имеют меньшее/большее значение чем в точке максимума/минимума. Этим такие точки отличаются от максимума/минимума вообще. В функции х³-х игрек может быть каким угодно большим, но минимум и максимум у нее в районе х=-/+0.5774.
Находятся они по значению производных. Берем производную. Она будут равна 3х²-1=у. Подставлем у=0, так как в этих точках функция обращается в ноль и не изменяется, и находим корни уравнения. Они равны +/-0.5774.
Но ты не знаешь какое именно число отвечает максимуму какое минимуму. Для этого берем производную от производной и подставляем туда найденные значения. Она равна у=6х. При подставленных найденых корнях производная будет отрицательной при отрицательном значении корня и положительной, растущей, при положительном. Знак найденного корня никак не показывает минимум это или максимум. Для наглядности найди критические точки у функции х³-х²-х.
#254 #345666
>>345659

>Сами её методы - хуйня, переходящая во что-то осмысленное только в алгебраической форме.


У тебя получилось объяснение в духе вербитопомазанного: хуйня - просто потому что не алгем, и не то, что нам велит любить Миша.
Но, на самом деле, ты же ничего не пояснил. Почему её методы хуйня безотносительно алгебраической формы?

>Зачем людям знать о таком недоразумении, как Евклид и о его работах


Хотя бы потому, что он первый начал использовать аксиоматически построенную теорию. А такого подхода придерживается сейчас вся современная математика.
>>345770
#255 #345667
>>345444

>аналогичным образом.


Точнее. Каждый выбросил одну и взял треть остатка?
#256 #345668
>>345651
Технарик закукарекал
>>345726
#257 #345669
>>345628
Если ты даже этого >>345630 не можешь понять, то ты реально даун и тебе надо было в пту съебывать после 9 класса.
#258 #345671
>>345657
Если функция в какой-то точке имеет экстремум, то в этой точке производная равна нулю. Поэтому чтобы найти наибольшее значение, тебе нужно вычислить значения функции во всех точках, где производная равна нулю и на границах области определения (если она ограничена) и выбрать максимум.
23 Кб, 684x515
#259 #345689
Ну и где эта ваша математика теперь?
113 Кб, 535x449
#260 #345695
>>345696
#261 #345696
>>345695
Что это значит?
>>345697
#262 #345697
>>345696
Это мемас.
>>345698
#263 #345698
>>345697
Ты пикчу >>345689 поясни, как так получилось? А то я веру в математику утратил.
>>345702>>345703
#264 #345702
>>345698
Потому что там не 2 = 4, а 0 = 0.
#265 #345703
>>345698
Ты видмо не понимаешь всей глубины проблемы. Очевидно: любое произвольное значение умножить на (0) = любое другое произвольное значение умножить на (0). (0) сокращаем так как он в скобках, следовательно любое произвольное значение = любое другое произвольное значение. То есть даже бесконечность = 1, а 1 = 0, следовательно бесконечность = 0. Такие дела.
#266 #345704
>>345113
Всегда получалось. А физика, биология и психология - лженауки. Нахуй все, оставьте только математику. Ахахаха!!!! Глупый шкальнек!!! Весь мир будет у ног математиков!!!
меня понесло
#267 #345705
>>345505
>>345507
Поехавший
>>345717
#268 #345717
>>345705
Нет. Дурак. Пробей в поиске.
#269 #345726
>>345668
Какое понятие ты считаешь неясным?
#270 #345728
>>345641
Пацаны, накидайте баек про ебанутых советских математиков. Давайте сравнивать ебанутость на международном уровне!

Рассказ очевидца с mathoverflow про дуэль арнольда с французами в 2001-м. Из арнольда бы хороший двачер получился

I was there. Arnol'd is one of my big mathematical heros, but I found the whole thing really sad. It was in French, but my French is decent. Arnol'd began his part with a phrase I've heard him say before: “In Russia it is impolite to talk ill of the dead, so I will not talk about Bourbaki” and then he proceeded to lambast Bourbaki in as nasty a way as you've ever read in any of his writings. I just felt like hanging my head. It was embarrassing watching him insult French Mathematics in front of 500 French men and saying things that seemed silly. It went on from there, Serre with a kind of sad understated dignity, not fighting, Arnol'd wanting a fight, hurling insults. The two barely even addressed each other. And yes, he did mention Toth, and if memory serves, he stated that Toth was probably Thales and had most likely come up with Newton's inverse square law. For me, the whole event was sad, embarrassing, and myth-crushing. Well, us mathematicians, we are all humans.
#271 #345736
>>345728
Математики этой страны были ебанутыми задолго до советов.

>еще в 20-х гг. консервативные механики вроде Чаплыгина пренебрегали этими новыми науками, считали их западной чушью. П.С.Александров рассказывал мне, что Чаплыгин запретил П.Урысону включать новую тогда общую теорию относительности в его аспирантский экзамен. Это - наша специфическая русская черта - склонность к консерватизму, к отрыву от мировой науки. Даже Чебышев в XIX в., при своем блестящем аналитическом таланте был патологическим кон­серватором. В.Ф.Каган рассказывал, что будучи молодым приват-доцентом он встретил старого Чебышева, пытался поведать ему о современной геометрии и т.д., а тот презрительно высказался о новомодных дисциплинах типа римановой геометрии и комплексного анализа. Созданная им школа была сильной, но и с сильной склонностью к провинциализму.


http://aspirant.rggu.ru/article.html?id=50768

Это эссе тоже ебанутый человек написал, кстати, - Сергей Петрович Новиков. Боговерчик, ватник и просто интересная личность.
>>345745
2 Кб, 207x219
#272 #345737
>>345689
это как-то совсем толсто.
>>345748
#273 #345743
>>345628
Мне бы твои проблемы. Я вот смысла частных производных никак понять не могу.
>>345747
#274 #345745
>>345728
>>345736
Идите-ка в /b со своими охуенными историями.
>>345766
#275 #345747
>>345743
Тоже самое что и ибычная производная, только там джве переменных.
Ближайший аналог это системы линейных уравнений. Там вроде тоже нихуя не понятно, но если выразить икс через игрек, то система вполнетрешается. Тут так же, только одна-две-N реременных считаеются постоянными, вокруг которых вычисляется одна свободная переменная.
#276 #345748
>>345737
Тоже толсто
37 Кб, 1825x148
#277 #345749
>>345747
Опять ты сука?
>>345750>>345751
24 Кб, 1890x155
#278 #345750
>>345747
>>345749
Алсо, вот это тоже твой пост. Ты уже не первый тред засираешь своей тупостью, я специально скриню.
>>345751
#279 #345751
>>345749
>>345750
Не знаю зачем ты скринишь всякую хуету, но я не он.
По существу есть что ответить про частные производные или нет?
>>345752>>345753
#280 #345752
>>345751
Вот тебе ссылочка и виктпедии, чтобы ты хоть как то мог связать свой поток сознания.
#282 #345754
>>345747
Частная производная - это производная вдоль вектора, когда в качестве вектора рассматривается вектор из наперёд выбранного зафиксированного базиса.
>>345755
#283 #345755
>>345754
А интеграл это площадь под кривой в пределах оси икс. Дальше что? Определение как суммы бесконечно малых неправильное?
>>345756
#284 #345756
>>345755
Написать тебе определение производной вдоль вектора?
>>345759
#285 #345759
>>345756
Не съезжай с темы.
Чем конкретно математическое, без маняпространств, определение частной производной написанное мной так, чтобы даже конченый смог понять, тебя не устраивает?
>>345760
#286 #345760
>>345759
Ты не говоришь, что такое частная производная.
Ты говоришь всего лишь о синтаксических преобразованиях.
Ты описываешь операцию, которую следует применять к строке символов, чтобы получить другую строку символов.
Это тупо.
Тебя спрашивают о смысле строк символов, а не о синтаксических манипуляциях.
Поэтому частная производная - это производная вдоль вектора из наперед заданного базиса.
Точка.
Свои "одна-две-N реременных считаеются постоянными" засунь себе в жопу.
#287 #345762
>>345760
Школореформатор, ты?
>>345764
#288 #345763
>>345760
Ты по ссылочке то хоть ходил?
Не буду с тобой больше спорить, все равно что об стену горох.
#289 #345764
>>345762
Я не хочу реформировать школы, я хочу убить всех методистов.
#290 #345766
>>345745
а весь бред, что выше-ниже, ничего? уровня sci?
#291 #345768
>>345760
нет, не про пикрил, а просто про интересную личность. да, книжица эта пиздец ацкий, выкини ее нахуй. возьми какой-нибудь нормальный вводный учебник для инженеров, желательно на английском языке. вот на таком примерно уровне

https://www.cengagebrain.com.au/content/9781285233260.pdf
>>345769>>345777
#292 #345769
>>345645
>>345768
сорян промазал
#293 #345770
>>345666
Аксиоматика в стиле Евклида это куча безумных аксиом и тонна работы, которая была доделана только Гилбертом. И после этого ты еще нихуя даже не сможешь доказать, ибо методы сильно нетривиальные.
Базовая же линейная алгебра оперирует только важными и простыми понятиями, из которых все очевидно выводится. Сейчас одного семестра линейной алгебры хватит, чтобы узнать, понять и научиться решать больше, чем любой великий геометр древности
#294 #345773
>>345728
Собственно, вся суть Арнольда. Прилюдно обмазывается говном, а все терпят из уважения к научным результатам.
>>345774
#295 #345774
>>345773

>а все терпят


А все получают особое, извращённое эстетическое удовольствие, как от просмотра хорошего копро-порно.
#296 #345775
>>345770
Ну и как гильберт обосновал постулат о параллельных прямых?
>>345776
#297 #345776
>>345775
Принял первый признак равенства треугольников за аксиому.
>>345778
#298 #345777
>>345768
Толсто. Такая хуйня не нужна математикам, потому что там акцент на ручном решении кучи дифуров. Такая хуйня не нужна инженерам, потому что в 2016 году есть мат по, в которое вбиваешь дифур и он решается.
>>345779>>345780
#299 #345778
>>345776
Собираешься своей массой пронзить пространство и время?
#300 #345779
>>345777
ну так то да, тогда и калькулюс нахуй не нужен. тогда нужно уточнить, что именно анон хочет получить в итоге.
#301 #345780
>>345777
Надто товсто.
Это все нужно для формирования мышдения. Ну и чтобы в случае ядерной войны можно было сразу создать интегральное исчисление, а не разбивать круг на отрезки нулевой длинны для нахождения пи.
>>345781>>345785
#302 #345781
>>345780
да нухй вообще инженерам мышление, есть же компьютеры. нужно просто обучать ремеслу - какие кнопки нажимать
>>345782
#303 #345782
>>345781

> Это все нужно для формирования мышдения.


> Ну и чтобы в случае ядерной войны



>>345781

> да нухй вообще инженерам мышление



Давайте все инженеры будут 3 семестра в шахматы играть и еще 2 в шашки, потому что это развивает мышление.
>>345783>>345790
#304 #345783
>>345782
1-й семестр - задачки решать про торговца шапками и фальшивую купюру
56 Кб, 468x600
#305 #345785
>>345780

>нужно для формирования мышдения


Нефальсифицируемый лозунг.
>>345793
#306 #345786
>>345770

>Сейчас одного семестра линейной алгебры хватит, чтобы узнать, понять и научиться решать больше, чем любой великий геометр древности


И кто тебя сейчас заставляет учить евклидову геометрию? На мат.факах нет такого предмета. Геометрия есть - только дифференциальная. Такую геометрию Миша вам не запрещает?
>>345788>>345791
#307 #345787
>>345770
И да, ты не ответил на вопрос, чем плоха евклидова геометрия безотносительно алгебраической формы?
#308 #345788
>>345786

>На мат.факах нет такого предмета


Есть на всех матфаках страны, кроме одного. Называется "аналитическая геометрия".
>>345789
#309 #345789
>>345788

>Есть на всех матфаках страны


Ты мат.фак со своей 247-й школой не путай.
>>345792
#310 #345790
>>345782
Как шашки математическое мышление сделают?
>>345796
#311 #345791
>>345786
Да никто не заставляет. Ты попросил объяснить, почему она говно и ненужна, я объяснил.
Я не тот анон, так что не знаю, что подразумевалос под алгебраической формой.
>>345798
#312 #345792
>>345789
что такое 247-я школа? УO? short bus?
#313 #345793
>>345785
Тобишь, у тебя пасанчик с падика и гаусс/бернулли имеют одинаковое мышление?
>>345804
#314 #345796
>>345790
А как решение дифуров и интегралов математическое мышление сделает?
>>345799
#315 #345798
>>345791

>Ты попросил объяснить, почему она говно и ненужна, я объяснил.


Что ты объяснил? Вот это:

>Аксиоматика в стиле Евклида это куча безумных аксиом и тонна работы


При желании, это можно сказать о любой аксиоматике. Да и чем аксиомы Евклида безумнее аксиом, скажем, ZFC?
>>345810
#316 #345799
>>345796
Не решение, а понимание сути как они работают.
А суть лучше всего понимается если их решать максимально подробно. Это не то же самое что просто решать.
Просто решать это написать что интеграл х² равен х³/3. Подробно решать это вывести почему интеграл х² равен х³/3
>>345801>>345802
#317 #345801
>>345799

> Подробно решать это вывести почему интеграл х² равен х³/3


Зачем ему выводить этот интеграл? Вот, например, я первокурсник и прошел основную теорему матана. Какое дополнительное понимание мне даст вычисление интеграла от x^2?
>>345803
#318 #345802
>>345799
пределы нужно инженерам объяснять? а вывод формул дифферецирования через пределы? а пруфы?
>>345803
#319 #345803
>>345801
>>345802
Лично вам вдвоем незачем. Вы сюда просто за бугагашками пришли.
>>345805
#320 #345804
>>345793
Пусть предмет A горячий и белый, а предмет B холодный и чёрный.
По твоей логике, если бы предмет A был чёрным, то он был бы холодным.
>>345808
#321 #345805
>>345803
На вопрос отвечай. Конкретный же вопрос задал. Представь, что я первокурсник, который прочитал теорему и ее доказательство
https://ru.wikipedia.org/wiki/Теорема_Ньютона_—_Лейбница
Что мне должно дать вычисление интеграла от x^2?
>>345806>>345808
#322 #345806
>>345805
Так ведь ты затем и пришёл в ВУЗ, чтобы изучить теорему Ньютона-Лейбница. Ты знал, куда шёл, и знал (должен был знать), чему тут будут учить. Уж не знаю, зачем тебе понадобилось изучать это.
>>345809
#323 #345807
реформист в треде, всем в методички
#324 #345808
>>345804
Не подменяй понятия. Еще раз спрашиваю, пасанчик с падика на одной волне с перельманом или нет?

>>345805
Ты чиьаешь плохо? Я уже ответил что тебе ничего. Разве что поможет корочки получить и пойти потом капчевать не по специальности с офиса за 20к, или сколько там офисные отбросы получают сейчас!?
>>345809>>345811
#325 #345809
>>345806

> Так ведь ты затем и пришёл в ВУЗ, чтобы изучить теорему Ньютона-Лейбница.


Ну, верно, я ее изучил, понял как считать интегралы. А интеграл от x^2 нахуя считать?

>>345808
Поздравляю, тебе получилось затроллить меня своей тупостью, у меня реально бомбит.
>>345812
#326 #345810
>>345798
Блджадь, в случае линала это нихуя не тонна работы и совершенно не безумно. Я тебе за пять минут определю, что такое евклидово пространство, прямые и прочее говно через линал, начав с натуральных чисел. Каждое определение будет прозрачным и использующимся во всех областях математики (поле, векторное пространство, билинейная форма).
После чего любая классическая теорема по геометрии будет практически очевидной.
А теперь сравни с вот этим
https://ru.wikipedia.org/wiki/Аксиоматика_Гильберта
Вот честно скажи, ты их воспроизведешь? Они хоть где-то еще появляются?
И сравни потом, насколько проще и надежнее сослаться на линейность и несложный счет, чем хуярить кучу дополнительных построений?
>>345815>>345816
#327 #345811
>>345808
Напомню тебе, что это ты доказываешь, что если заставить пасанчика сыграть достаточное количество партий в шахматы, то он превратится в Пуанкаре.
>>345814>>345938
#328 #345812
>>345809
Еще раз повторяю, специально для контуженных с бронепоезда. Лично тебе знания сути работы интегрального исчисления ничего не дадут.
Не всем же надо знать как получается электричество в проводе, чтобы включить лампочку. Но вот когда провод где то замыкает...
>>345820>>345822
#329 #345814
>>345811
Ты подмениваешь понятия. Я писал про математический склад мышления, а не про стратегическое планирование. Не пиши мне больше.
>>345817
#330 #345815
>>345810
Определяй и доказывай. Назвался груздем...
#331 #345816
>>345810

>После чего любая классическая теорема по геометрии будет практически очевидной.


Серьёзно? Классические теоремы геометрии: теорема Наполеона Бонапарта, теорема Монжа, теорема Сальмона, теорема Микеля. Ты обломаешься уже на формулировках.
>>345847
#332 #345817
>>345814
Слив засчитан, лол.
>>345818
#333 #345818
>>345817
Как скажешь.
#334 #345820
>>345812
Сука, ну какой же ты даун. Я знаю суть работы интегрального исчисления, понимаешь? Я могу вычислить интеграл от многочлена. Вопрос в том, нахуя мне вычислять интеграл от x^2?
>>345822
#335 #345822
>>345820
Я высказал тут >>345812 все что хотел и не буду повторять все трижды.
Могу только добавить что если ты, таки да, знаешь как и что работает, ты бы расписал как получается интеграл из функции квадрата. Все.
>>345830
#336 #345830
>>345822
Я тебе объясню ход мыслей нормального человека.

1) Многочлен - непрерывная функция. Непрерывные функции интегрируемы.
2) Из линейности интеграла следует, что
integral sum a_k x^k = sum a_k integral x^k
3) Интеграл от x^k можно посчитать по формуле Ньютона-Лейбница. Легко догадаться, что первообразная x^k - это x^(k+1) / (k + 1).

Ход мыслей долбоеба: ну кароч надо вычислить интеграл от sin 666x + x^1488, чтобы понять как работает интегрирование))0
>>345840
#337 #345838
>>345833
Где?
#338 #345840
>>345830
Вот ты и показал свой уровень владения дискуссией и математикой. Никаких доказательств и обоснований, только три предложения, которые каким то магическим образом вместили чуть ли не полторы тысячилетия развития математики, как тебе кажется.
Я рад что больше мне не придется с тобой разговаривать.
>>345843
#339 #345843
>>345840
Все, что я написал, общеизвестно. Если ты не можешь доказать что-то из этого или не знаешь, что именно надо гуглить, то ты просто даун. Это все проходят на 1 курсе.
>>345946
#340 #345844
Саентач, как же у меня горит от моей тупости!

Вот есть область целостности R, а есть элементы x1,...,xn, которые ей принадлежат. Так вот, говорится, что если dR=x1R+...+xnR имеет место, то d=gcd(x1,...,xn). И все бы ничего, но я, блиа, никак не могу понять как делитель может равняться сумме того, что он делит. Алсо, кажется мне, что я не правильно понимаю смысл написанного. Помогай, помогач!
>>345848
#341 #345845
Пацаны, я чет запутался, кто в этом треде самый умный труъ, покажите его посты? Просто хочу отсечь, все кто с ним спорит - говноеды, и их игнорировать
>>345846
#342 #345846
>>345845
Ты. Теперь умолкни навсегда.
>>345854
#343 #345847
>>345816
Все кроме Салмона совсем просто. Особенно, Бонапарт насмешил, на фикипедии говно написано геметрическое, хотя всем очевидно, что через комплексные числа это делается за секунду.
>>345849
#344 #345848
>>345844
Не сумме, а линейной комбинации. Там могут быть отрицательные коэффициенты. Никогда не слышал про алгоритм Евклида?
>>345852
#345 #345849
>>345847

> через комплексные числа это делается за секунду


Сделай, покажи класс.
>>345857
#346 #345850
>>344306
Это не переиздание? И кто ныньче в группе, если так?
>>345855
#347 #345852
>>345848
Увы, не слышал. Математикой увлекся недавно. Мне почему-то хочется d назвать общим кратным x1,...,xn. Я совсем конченный? x1R,...,xnR - это, вроде, идеалы. Это что-нибудь меняет?
#348 #345853
>>344310
наполеона сообщник
#349 #345854
>>345846
Ну я серьезно, здесь лайки нужны, + -. А так все анонимные умники.
>>345856
162 Кб, 1028x746
#350 #345855
>>345850
Судя по тому, что в книге используется теория категорий, - явно кто-то молодой, резвый, наглый.
>>345866
#351 #345856
>>345854
Анус себе лайкни.
>>345860
#352 #345857
>>345849
Поворот на 60 градусов = умножение на корень шестой степени из единицы. Центр правильного треугольника = сумма вершин делить на три. Так что хуяришь начало координат как тебе удобнее и делаешь пару-тройку тривиальных арифметических действий.
>>345861
#353 #345859
Похоже, скоро на пикче с парашей придётся рисовать дедекиндовы кольца рядом с N.
#354 #345860
>>345856
А чому себе, а не тебе, например?
#355 #345861
>>345857

>Поворот на 60 градусов = умножение на корень шестой степени из единицы. Центр правильного треугольника = сумма вершин делить на три. Так что хуяришь начало координат как тебе удобнее и делаешь пару-тройку тривиальных арифметических действий.


И что за говно ты высрал, кокормист? Вычисли координаты равносторонних треугольников, построенных на сторонах произвольного треугольника относительно какой хочешь системы координат.
>>345865
#356 #345865
>>345861
Пиздос, ты ебанутый или как?
Треугольник ABC, пускай 0 в А. Пусть B и С задаются комплексными числами z_1,z_2. Тогда AB достраивается числом z_1w, AC достраивается числом z_2(w^-1), BC достраивается числом z_1+(z_2-z_1)*w. Где w первообразный корень шестой степени из 1. Теперь очевидно считаешь центры и проверяешь, что разности отличаются домножением на w.
#357 #345866
>>345855

>теория категорий


>явно кто-то молодой


Ты ебанулся, категории уже с 40-хх годов активно используются.
>>345867
#358 #345867
>>345866
Ты явно не знаком с историей группы Бурбаки.
>>345868
#359 #345868
>>345867
Зато я знаком с историй теории категорий.
>>345869
#360 #345869
>>345868
Тут все с ней знакомы, кроме школьников.
>>345871
#361 #345871
>>345869

>все



Да тут не считая мимокрокодилов человек 5 постит по-моему. И во всем саентаче один и тот же стиль, то есть те же люди. Сколько тут всего народу, человек 20 в хороший день?
>>345872
#362 #345872
>>345871
355 | 32 | 100

>100 постеров

>>345873>>345886
#363 #345873
>>345872
Но это 90% мимикроки? А костяк так сказать, стержень, это 2-3 мегапостера, которые друг с другом срутся?
>>345874
#364 #345874
>>345873
И тем не менее каждый мат.тред набирает минимум по сто постеров до бамп лимита. Хотя меня тоже это удивляет, кто все эти люди? Может Абу постеров накручивает, чтобы придать вес доске.
>>345875
#365 #345875
>>345874
Ну та, хз что это за статистика, может с проксями мутят
#366 #345882
Так дауны, кто не сможет ответить на следущий вопрос с доказательством того я рот ебал нахуй:

Все ли нетривиальные нули дзета-функции лежат на прямой Re(z)=1/2?
>>345883>>345884
#367 #345883
>>345882
Все.
>>345885
#368 #345884
>>345882
Риман, замолчите, вы умерли. Обратитесь в институт Клэя.
#369 #345885
>>345883
ну спорно, может и не все, а просто не нашли еще опровергающего алгоритма. всего то требуется найти решение для интеграла через численные методы, а никто не ищет, ибо компьюетрных мощностей не хватает. можно через хадуп попробовать.
>>345887
#370 #345886
>>345872
как сто, я вижу 96. это шо такое пацаны? выпиливают постеров?
#371 #345887
>>345885
Когда найдешь тогда и поговорим(то есть никогда)
#372 #345924
Идите в хинтай, умники
#373 #345930
>>345078
И ничего он не ебанут, просто биологию не знает. Математичен ли мир - не известно. Но вот все восприятие мира - это сплошная математика. Вот взять зрение. Мы же не видим фотоны. Фотоны падают на группы нейронов сетчатки, те - генерируют сигнал. В мозге все сигналы обрабатываются, например, выявляются границы объектов. Вы же понимаете, что нет таких фотонов, которые несут информацию "я означаю границу этого объекта" независимо от наблюдателя.
>>345932
#374 #345932
>>345930
Причем тут биология?
6 Кб, 259x194
#375 #345936
Ты дифференциируем. Ты в гильбертовом пространстве, и я в гильбертовом пространстве. А потом сходимость в нашу честь... Сходимость в нашу честь. Сначала преобразуемся по Коши, потом интегрироваться пойдём. Производные равны нулю будут. Градиент. Сходимость. В мою честь сходимость. Я интегрируемый, и ты - в гильбертовом пространстве. Посмотри, какая у меня топология! Посмотри, какие у неё диффеоморфизмы! Я её ещё никому не показывал. Только тебе покажу. Посмотри! Это моя парадная. Специально вычислял! Посмотри, не бойся! Симплексы? Я их поменяю! Вот они, симплексы. Вот! Настоящие! Видишь? Математик. Я математик. Я их на преобразование по Коши надену.
#376 #345938
>>345811
Пуанкаре стал гением потому, что переболел дифтериеей типо же
#377 #345946
>>345843
Кокая мощная аргументация. Сразу видно что после ядерной войны сможешь с нуля построить всю высшую математику.
#378 #345947
#379 #345949
>>345946
Этого не понадобится
>>345950
#380 #345950
>>345949
--> /re
>>345951
#381 #345951
>>345950
Нет, просто после ядерной войны эта хуита совсем бесполезна будет.
>>345953
#382 #345953
>>345951
ОЛОДО РЕЙДЕРИ АНАНЕМ ЛЕВГЕОН НАБИГАЕМ ДОМИКИ ДЕРЕВЯНЫЕ МОЖНО ГРАБИТЬ КОРОВАНЫ
>>345955
#383 #345955
>>345953
Ты поехал
>>345957
#384 #345957
>>345955
--> /b
#385 #346016
Аноны, вектор как n-tuple в некотором векторном пространстве (с соответсвующими операциями и аксиомами разумеется) - это достаточно абстрактное определение вектора? Или в еще более общеем случае вектор это не n-tuple? А что тогда?
>>346019>>346065
#386 #346019
>>346016
Элемент векторного пространства.
>>346022
#387 #346021
тест
#388 #346022
>>346019
Так я и спрашиваю - в каком векторном пространстве элемент не n-tuple? Или наоборот, над каким полем должно быть определено в.п. чтобы элемент считался n-tuple? Что-то про координатные системы?
#389 #346028
>>346022
А, вот нашел ответ. Все-таки вектор это прежде всего геометрическое понятие, а все остальное - булшит.

The proper treatment of calculus for functions of several
variables requires vector ideas; the budding statistician and
the coming physicist need them; modern analysis is unthink-
able without the notion of linear dependence and all that flows
from it. Throughout these courses the infusion of a geomet-
rical point of view is of paramount importance. A vector
is geometrical; it is an element of a vector space, defined
by suitable axioms—whether the scalars be real numbers or
elements of a general field. A vector is not an
n
-tuple of
numbers until a coordinate system has been chosen. Any
teacher and any text book which starts with the idea that vec-
tors are
n
-tuples is committing a crime for which the proper
punishment is ridicule. The
n
-tuple idea is not ‘easier,’ it is
harder; it is not clearer, it is more misleading. By the same
token, linear transformations are basic and matrices are their
representations
>>346050
#390 #346050
>>346022
В бесконечномерном пространстве вектор - не n-tuple. Бесконечномерные пространства встречаются очень часто. Например. R - бесконечномерное пространство над полем рациональных чисел. Но базис этого пространства не то что не конечен - он даже несчётен.

Конечномерные векторы превращаются в n-tuple только после фиксации в векторном пространстве какой-то системы координат. В двух разных системах координат одна и та же n-tuple может соответствовать двум разным векторам, один и тот же вектор может иметь две разные n-tuple. Вектор и n-tuple - разные вещи. Правильнее думать, что над каждым полем K для любого натурального числа n существует "модельное пространство энок", и что любое пространство над K с dim = n изоморфно модельному пространству n-ок, причём многими разными способами.

>>346028
Вектор - это элемент векторного пространства. Векторное пространство - это абелева группа, на которой действует поле. Абелева группа - это движения какого-то объекта.
>>346084
#391 #346055
>>345946
Всю не всю, но матан на уровне 17 века смогу.
#392 #346056
>>346022
Возьми, например, пространство последовательностей в R. Понятно, я думаю, как складывать последовательности друг с другом и как умножать их на скаляр.
#393 #346059
сап, анон. Возможен ли факториал числа с плавающей точкой?
>>346061
#394 #346061
>>346059
Разумеется. Гамма-функция называется.
#395 #346065
>>346016
Достаточно для чего?
>>346073
#396 #346073
>>346065
Чтобы математики не доебывались, что ты не знаешь, что такое вектор.
#397 #346076
>>345728
Единственный смешной математический анекдот, который я когда-либо встречал:

Emil Artin (1898–1962) was famous for the extremely clear and
extraordinarily well presented lectures that he always gave without
any notes. One day, midway in a proof, he suddenly hesitated and
said: “this conclusion is trivial”. After a few seconds, he repeated:
“it is trivial, but I no longer know why”. He then thought about
the question for another minute and said: “I know that it is trivial,
but I no longer understand it”. He reflected on it a few moments
more and finally said: “excuse me, I have to look at my lecture
notes”. He then left the room and came back ten minutes later
saying: “it really is trivial”.
(Witnessed by Prof. Josef Schmid, Fribourg)
>>346081
#398 #346078
>>345397
Бамп, посоны.
#399 #346081
>>346076
У этого анекдота борода больше чем у Перельмана.
#400 #346084
>>346050
Ох ебта, надо неделю над этим думать.

Другими словами, определение вектора через n-tuple ни чем не "обобщеннее" определения вектора из школьной геометрии как отрезка прямой имеющего величину и направление? А просто его алгебраибическая интерпретация? То есть всегда из тюпла можно построить геометрический вектор, если "мерность" пространства позволяет конечно? Я думал n-tuple это прямо так абстрактно, что все.
>>346090>>346174
#401 #346090
>>346084

>всегда из тюпла можно построить геометрический вектор


Сначала нужно указать базис пространства. Одна и та же энка может быть координатами любого из векторов пространства, какого конкретно - зависит от базиса.

>это прямо так абстрактно, что все


Хочешь вменяемо определить векторное пространство - воспользуйся понятием "модуль над кольцом" и скажи, что векторное пространство - это модуль над полем. Поле - это частный случай кольца.
>>346095>>346388
#402 #346095
>>346090
Ну да, я имел ввиду R^n, эвклидово (или декартово?!) пространство со стандартным базисом. А что такое эвклидово пространство без указания базиса?
>>346107>>346343
#403 #346107
>>346095
Сколько тебе лет? Ответь честно.
#404 #346174
>>346084
Векторы в математике - это то, что ты можешь складывать и умножать на числа. Твои тюплы можно рассматривать как векторное пространство, но точно так же и функции на R (все/гладкие/полиномиальные/и.т.д), которые тюплами не являются, можно рассматривать как векторное пространство, потому что их можно умножать на числа и складывать.
#405 #346212
Как научиться в картофан: не получаются всякие огромные выражения, постоянно теряются какие-то знаки или скобки или числа. У кого есть такая проблема??
>>346225
#406 #346225
>>346212

>постоянно теряются какие-то знаки или скобки или числа.


Может ты даун. У них подобные проблемы. Проверься у доктора.
#407 #346242
Кто-нибудь когда-нибудь ещё пытался создать трактат по математике на подобие оного у Бурбаки? Что вышло? Можно ли почитать подобное?
#408 #346243
>>346242
Такое уже устарело. В наши дни люди пытаются формализовать математику полностью, чтобы кодить теоремы на каком-нибудь идрисе вместо описания их словами.
#409 #346245
>>346242
Рассел, но его трактат "Principia Mathematica" провалился. В наши дни формализацией математики занимается программистский проект с говорящим названием "Петух" (coq по-французски), успехов нет.

Книги Бурбаки - единственный успешный проект такого рода.
>>346254>>346256
#410 #346249
>>346242
Зачем, если есть Бурбаки. Подобные проекты не из воздуха появляются, нужно, чтобы назрела необходимость. Такого пока не произошло, хотя, по-моему, намечается. Конечно, появились компьютеры, по этому поводу уже сказали.
>>346250
#411 #346250
>>346249
Ах да, категории тоже появились, это один из ингридиентов слова «намечается». Но одних категорий маловато. Впрочем, человечеству скоро будет не до этого. Война (вариант — катастрофа, кризис, коллапс...) начнётся, закончится, тогда и будет Бурбаки 2.0, в соответствии с первой версией.
>>346273
97 Кб, 220x317
#412 #346252
>>346242
Смысл? На тех же основаниях получится то же, что у бурбаков, но хуже. Вот если бы кто-нибудь осилил изложение всей математики с позиции пикрелейтед, то могло бы получиться нечто отличающееся от бурбаков. Но вряд ли кто осилит.
#413 #346254
>>346245
Что еще раз показывает, что программисты - петухи.
#414 #346256
>>346245

>В наши дни формализацией математики занимается программистский проект с говорящим названием "Петух" (coq по-французски), успехов нет.


Кто тебе сказал, что успехов нет?
>>346258>>346259
#415 #346258
>>346256
Тогда почему его ещё не используют все чистые математики? В чём вообще может заключаться полезность "Coq"?
>>346374
#416 #346259
>>346256
Априорное знание и метаиндукция.
>>346262
#417 #346262
>>346259
Маняматик обезумел.
Сумеет ли галоперидол совладать с ним?
>>346263>>346264
#418 #346263
>>346262
In Soviet Russia математик овладевает галоперидол.
#419 #346264
>>346262
Почему? Разве он не прав?
>>346266
#420 #346266
>>346264
Он говорит что маняматикой надо заниматься ради маняматики, а это плохо. Априорного знания быть не моожет, яскозал.
Но конечно же тут он возразит что две пересекающиеся прямые в н-мерном пространстве лежат в одной плоскости, этажиачивидна.
Хуйню написал. да.
>>346272
knuebok #421 #346272
>>346266
So cute ^_^ Мы просто в разных дискурсах с тобой находимся, понимаешь?
>>346274
#422 #346273
>>346250

> появились категории


> 2016

#423 #346274
>>346272
Понимаю.
Ящитаю что занятие математикой ради математики это тупик и загнивание.
Тот же спор о струне принес больше в математику чем нынешние попытки натянуть теорию струн на реальность.
#424 #346290
ЗАЧЕМ НУЖЕН COQ?
>>346295
#425 #346295
>>346290
Для написания proof-assistant-ов.
>>346297
#426 #346297
>>346295
С ним можно вывести любую элементарную теорему?
#427 #346343
>>346095
Ну прочитай определение базиса. Например, (1, 1) и (-1, 1) - базис R^2.
>>346388
76 Кб, 546x390
19 Кб, 527x118
#428 #346362
Поясни за малые множества, Анон. Что это такое? Точнее, чем категория Sets отличается от Ens(от какого слова взято это сокращение?)? Как в Sets так и в Ens определяется некий "универсум", значит разница в количестве стрелок?
>>346364>>346486
#430 #346374
>>346258

>Тогда почему его ещё не используют все чистые математики?


Потому что не все могут в него. Те, кто может - используют. Ты про гомотопическую теорию типов и Воеводского слышал?

>В чём вообще может заключаться полезность "Coq"?


В машинном доказательстве теорем, во-первых. Потом, написание кода, работающего строго по заданным спецификациям, т.е. программное обеспечение без глюков и ошибок. Ты и этого не знал?
#431 #346388
>>346090

>Например, (1, 1) и (-1, 1) - базис R^2.


Ну это я кажись понимаю. Берем стандартный базис (1,0) (0,1) и поворачиваем на 45 CCW?

Да не пацаны, я хоть и школьник, но не совсем даун (кажется), я понимаю что такое базис (по-крайней мере в координатном пространстве), я просто похоже хуйню спросил, потому что заморочился >>346343

>сначало нужно задать базис



и споткнулся я об "задать". т.е. является ли базис как бы "intrinsic property" любого в.п. В результате тупо через гугл
"every vector space has a basis" -- 9000 хитов.
Успокоился.
>>346390
#432 #346390
>>346388
Любое конечномерное векторное пространство однозначно определяется размерностью n и поэтому изоморфно этим твоим тьюплам K^n, в этом смысле никаких (конечномерных) векторных пространств кроме K^n нет. Просто изоморфны они не одним способом - это и есть выбор базиса.
#433 #346393
>>346390
Че за хуйню ты несешь, поехавший? Школьник, не слушай его.
>>346395
#434 #346394
>>346390
Кстати а поясни по хардкору за эти мои любимые тюплы.

>Конечномерные векторы превращаются в n-tuple только после фиксации в векторном пространстве какой-то системы координат.



Что есть в твоем понимании n-tuple?
Как я понимаю это же просто абстрактный упорядоченный список каких угодно элементов. Вот есть у меня какое-то абстрактное в.п. уровня /b.
То есть в нем - хоть пики точеные, хоть хуи дроченые. А базис я вообще не знаю какой.
Разве не может у меня быть тюпл дроченых хуев? Он же вектор?

тюпл дроченых хуев - это охуенный подарок был бы EOT на 8 марта
>>346395>>346397
#435 #346395
>>346393
Ты ебанут? Всё правильно он сказал. Для любого целого положительного числа n и любого поля K есть только одно векторное пространство размерности n над полем K, с точностью до изоморфизма.

>>346394
Я не он, но поясню. Хуи должны образовывать абелеву группу. Ты должен быть в состоянии сложить два хуя и для каждого хуя найти минус хуй.
>>346396>>346400
#436 #346396
>>346395
Пацаны вы охуенны! Я копипастаю ваши ответы, и пойду гуглить пару дней. Потом мамке расскажу, какой я у нее математик.
#437 #346397
>>346394
Открой учебники и прочитай нормальные определения, зачем ты хуйней страдаешь?

n-tuple - это элемент декартова произведения каких-то n множеств.
Второй вопрос я не понял.

Алсо, я не он >>346390 , он вообще пиздец отбитый.

> Любое конечномерное векторное пространство однозначно определяется размерностью n


Тогда бы говорили, что векторное пространство - это натуральное число.
#438 #346398
>>346397
У меня учебников хоть жопой жуй. Постоянно натыкаюсь на "подразумеваемые понятия". Толкования требуют как тора с талмудом. n-tuple видимо имеет разные определения в разных областях
>>346400
#439 #346400
>>346398
https://ru.wikipedia.org/wiki/Кортеж_(информатика)#.D0.92_.D0.BC.D0.B0.D1.82.D0.B5.D0.BC.D0.B0.D1.82.D0.B8.D0.BA.D0.B5
Вот здесь все понятно написано. Кортеж это и есть tuple.

>>346395

> Для любого целого положительного числа n и любого поля K есть только одно векторное пространство размерности n над полем K, с точностью до изоморфизма.


Возьмем пространство, у которого вектора из R^n, а скаляры из Q и возьмем пространство, у которого вектора из Q^n, а скаляры из Q. Они не могут быть изоморфны, потому что в первом пространстве несчетное число векторов, а во втором - счетное.
>>346402>>346403
#440 #346402
>>346400
Упс, я объебался, R^n над Q бесконечномерное.
#441 #346403
>>346400

>у которого вектора из R^n, а скаляры из Q


Вот здесь у тебя проблема. Ты можешь взять произвольную абелеву группу и рассмотреть её как модуль над угодным тебе полем, с этим проблем нет. Rn образует абелеву группу, это тоже верно. Но кто сказал, что размерность Rn над полем Q будет равна n? Она не будет. Уже размерность R над Q не только не конечна, но даже несчётна.
#442 #346407
Как научиться coq'у?
>>346408
#443 #346408
>>346407
Зачем тебе учиться Петуху?
>>346410
#444 #346410
>>346408
Я N-петух.
#445 #346412
>>346397

>Тогда бы говорили, что векторное пространство - это натуральное число.


Что?
>>346413
#446 #346413
>>346412
>>346397

>Тогда бы говорили, что векторное пространство - это натуральное число.


Вообще-то вопрос осмысленный. Разница в том, что у натуральных чисел нет нетривиальных эндоморфизмов, а у векторных пространств они есть.
>>346415>>346439
#447 #346415
>>346413
Если выбрать каноничных представителей для каждой размерности и каноничный набор сюрьекций и вложений между ними, то получим, что категория конечномерных векторных пространств и линейных отображений между ними расслоена в групоидах над категорией симплексов - см.
https://ncatlab.org/nlab/show/simplex+category
и https://ncatlab.org/nlab/show/fibration+fibered+in+groupoids
>>346437
#448 #346421
Реквестирую литературу по теории групп (интересуют группы Ли), если будет книга с со спецификой физики конденсированного состояния. Могу и на английском, желательно, посовременнее, наверно. Имею математическую базу физического факультета.
>>346429>>346434
#449 #346429
>>346421
Вроде бы есть Монастырский М.И. Топология калибровочных полей и конденсированных сред. Судя по оглавлению, необходимый бэкграунд там излагается, судя по названию, оно по теме. Больше сказать не могу
>>346433
#450 #346433
>>346429
Спасибо, пока гуглил, наткнулся на любопытные обсуждения.
#451 #346434
>>346421
Есть у Рубакова две книжки называются "Классические калибровочные поля". Там есть кое-что, но приложения тривиальные в духе спонтанного нарушения симметрии. Тем не менее весь групповой формализ, который ты встретишь в кондесированных средах там присутствует кроме, может быть, обсуждения конформной группы.
#452 #346437
>>346415

>набор сюрьекций и вложений


Почему у сюръекции нет удобного аналога? Типа "всеобразие".
>>346438>>346440
#453 #346438
>>346437
Иногда говорят отображение в/на, но это тоже не всегда удобно.
#454 #346439
>>346413

> у натуральных чисел нет нетривиальных эндоморфизмов


Если ты хочешь чтобы это соответсвие было функториально, то есть.
>>346441
#455 #346440
>>346437
Наложение.
#456 #346441
>>346439
Ну да, наверное. Но вот автоморфизмов по-моему все таки нет (или есть?).
>>346447
#457 #346447
>>346441
Я что-то не улавливаю.
Если ты хочешь чтобы соответствие натуральные <-> конечномерные пространства было функториально, то категория целых должна просто копироваться с категории векторных пространств, таким образом морфизмы в этой категории это множество матриц n x m. Автоморфизмы из n в n это обратимые n x n матрицы.
>>346451
#458 #346451
>>346447
Я и не говорил, что строю соответствие между двумя категориями, наоборот я хотел показать, что это разные категории. Если определить категорию натуральных чисел как пишешь ты, то безусловно они будут эквивалентны. Но первоначальное утверждение было

>Тогда бы говорили, что векторное пространство - это натуральное число.


и я хотел продемонстрировать, чем же конкретно отличается категория векторных пространств от натуральных чисел.
А именно: симплициальная категория имеет в качестве объектов множество натуральных чисел, а в качестве морфизмов неубывающие отоьражения множеств {0,1,...,m}->{0,1,...,n}. Все эти морфизмы можно разложить в композицию двух типов отображений n->(n-1) и n->(n+1), отвечающих проекции симплекса на i-ю грань или включению i-й грани в симплекс.
Тогда выберем набор 'каноничных' n-мерных пространств для каждого n и набор каноничных вложений и проекций R^n->R^(n+1) или R^n->(n-1), отвечающих вышеописанным отображениям. В таком случае любое линейной отображение можно представить в виде композиции обратимого отображения (приводящего базис к "каноничному") и композиции этих вложений/проекций.
Теперь сопоставим каждому в.п. его размерность, а каждому лин. отображению композицию соответствующих ему отображений включения грани/проекции на грань в симплициальной категории.
Тогда нетрудно видеть, что слой этого функтора (т.е. набор объектов, отображаемых в заданный объект и с набором морфизмов, которые отображаются в тождественный морфизм в категории-цели) над любым нат. числом есть категория n-мерных пространств с морфизмами-обратимыми отображениями. Эта категория является группоидом (это значит, что каждый морфизм в ней обратим). В свою очередь это значит, что первоначальная категория расслоена в группоидах (на самом деле только pre-fibered, потому что нужно выбрать cleavage, но это детали).
Таким образом разница между категорией в.п. и категорией нат. чисел состоит в наличии в первой у каждого объекта нетривиальных автоморфизмов - что я и хотел показать.
#458 #346451
>>346447
Я и не говорил, что строю соответствие между двумя категориями, наоборот я хотел показать, что это разные категории. Если определить категорию натуральных чисел как пишешь ты, то безусловно они будут эквивалентны. Но первоначальное утверждение было

>Тогда бы говорили, что векторное пространство - это натуральное число.


и я хотел продемонстрировать, чем же конкретно отличается категория векторных пространств от натуральных чисел.
А именно: симплициальная категория имеет в качестве объектов множество натуральных чисел, а в качестве морфизмов неубывающие отоьражения множеств {0,1,...,m}->{0,1,...,n}. Все эти морфизмы можно разложить в композицию двух типов отображений n->(n-1) и n->(n+1), отвечающих проекции симплекса на i-ю грань или включению i-й грани в симплекс.
Тогда выберем набор 'каноничных' n-мерных пространств для каждого n и набор каноничных вложений и проекций R^n->R^(n+1) или R^n->(n-1), отвечающих вышеописанным отображениям. В таком случае любое линейной отображение можно представить в виде композиции обратимого отображения (приводящего базис к "каноничному") и композиции этих вложений/проекций.
Теперь сопоставим каждому в.п. его размерность, а каждому лин. отображению композицию соответствующих ему отображений включения грани/проекции на грань в симплициальной категории.
Тогда нетрудно видеть, что слой этого функтора (т.е. набор объектов, отображаемых в заданный объект и с набором морфизмов, которые отображаются в тождественный морфизм в категории-цели) над любым нат. числом есть категория n-мерных пространств с морфизмами-обратимыми отображениями. Эта категория является группоидом (это значит, что каждый морфизм в ней обратим). В свою очередь это значит, что первоначальная категория расслоена в группоидах (на самом деле только pre-fibered, потому что нужно выбрать cleavage, но это детали).
Таким образом разница между категорией в.п. и категорией нат. чисел состоит в наличии в первой у каждого объекта нетривиальных автоморфизмов - что я и хотел показать.
>>346452>>346519
#459 #346452
>>346451
Поправка

>в качестве объектов множество натуральных чисел


в качестве объектов множество неубывающих последовательностей нат. чисел

>отоьражения множеств


отображения множеств.
На самом деле здесь есть небольшая неточность - нам нужно рассматривать не категорию всех симплексов, а только категорию невырожденных симплексов, т.е. не являющихся гранью. Иначе придется иметь дело с "вырожденными векторными пространствами" которые имеют размерность меньше n, но должны бы считаться n-мерными следуя нашим определениям. Переход к невырожденным симплексам исправляет проблему.
#460 #346457
Ананасики, я даун. Что читать сначала чтобы разобраться в математике: Лэнга или ван дер Ваардена?
>>346458>>346460
#461 #346458
>>346457
Лэнга, он на пятьдесят лет новее и няшные категории есть.
>>346462
#462 #346460
>>346457
Не вкатишься. Проботай стандартный институтский курс вроде Кострикина или Винберга.

Математика = алгебра. Дебил, что ли?
>>346461>>346462
#463 #346461
>>346460
Алгебра является универсальным языком математики, что поделать.
2192 Кб, Webm
#464 #346462
>>346460

>Дебил, что ли?


-->

>Ананасики, я даун



>Проботай стандартный институтский курс вроде Кострикина или Винберга


Спасибо. Скачал Винберга. Количество учебников к прочтению начинает пугать, попробую читать все параллельно.

>>346458
Спасибо. Хотя мне до категорий еще как до Австралии.

Алсо, ловите чихающего тюленя.
>>346463
#465 #346463
>>346462
А смысл читать несколько? Во всех учебниках излагается стандартный материал. Ленг (современное издание, а не 1967 года) и Aluffi Algebra chapter 0 в этом смысле просто самые полные. Кострикин лично мне не нравится, слишком сильно вязнет в деталях и почти не излагает современные разделы. Винберг, Artin и т.п. undergraduate курсы примерно эквивалентны друг другу, можно читать любой.
Считать теорию категорий "продвинутым" предметом тоже по-моему неправильно - никаких содержательных результатов там нет, это просто язык современной математики. Все что реально может из нее понадобиться - несколько утверждений о сопряженных функторах и лемма Йонеды. Даже читать по ней какие-нибудь пособия я считаю излишним.
>>346464>>346467
#466 #346464
>>346463

>никаких содержательных результатов там нет


Нетривиальные результаты там есть; несвѣдь их.
>>346465
#467 #346465
>>346464
Смотря что считать нетривиальным результатом. Для меня это такой, который бы привёл к решению некой математической проблемы, не относящейся изначально к области теории категорий. Вот например методами теории моделей передоказали mordell lang conjecture. Хотелось бы услышать о чем-то подобном в теории категорий.
>>346466
#469 #346467
>>346463

>лемма Йонеды


А я ведь загуглил что это такое. Лол, вроде как это обобщение теоремы Кэли. Но если теорему Кэли я понимаю и, наверное, смогу доказать, то в формулировке этой леммы я даже слова в осмысленные словосочетания связать не могу.
>>346468
#470 #346468
>>346467
Во-первых, я не понял, при чем здесь теорема Кэли. Во-вторых, она же в уме доказывается.
Что такое функтор C->Set? Это отображение, ставящее в соответствие каждому объекту X из C множество F(X), каждому морфизму f:X->Y отображение множеств F(f):F(X)->F(Y) так, что сохраняется ассоциативность и единичный элемент. Представимый функтор имеет вид Hom(A,X) для некого объекта A (здесь используется малость С, т.к. Hom должен быть множеством). Пусть теперь дано преобразование Hom(A,-)->F, поставим ему в соответствие образ в F(A) тождественного морфизма id_A из Hom(A,A). Наоборот, пусть задан элемент a из F(A). Тогда данному g:A->X поставим в соответствие F(g)(a) из F(X). Легко проверить, что это удовлетворяет определению естественного преобразования, а также что эти две конструкции обратны друг другу. Вот и лемма Йонеды.
>>346469>>346503
#471 #346469
>>346468

>она же в уме доказывается


Предварительно нужно потратить некоторое время на понимание определений. А так да, ничего сложного.
#472 #346486
>>346362
Sets, категория всех малых множеств, не является малой категорией. Класс её объектов есть собственный класс, ибо. Ens инкапсулирует подмножества какого-то множества, не являющегося собственным классом (т.е., это не универсум во всяком случае).
#473 #346503
>>346468

>при чем здесь теорема Кэли


Википедия так говорит.

>она же в уме доказывается


Если ты разобрался в теории групп, а я только начал разбираться. Плюс я не знаю что такое функтор, что такое морфизм и т д, поэтому такое определение леммы для меня малоинформативно.

Пойду, короче, Винберга читать. Может поумнею.
>>346507
#474 #346507
>>346503
Морфизм - это стрелка между объектами категории. В разных категориях природа морфизмов различается. В Set это функции на множествах, в Grp гомоморфизмы, в Mat(K) это матрицы с коэффициентами из K. Функтор - морфизм целых категорий. Технически функтор состоит из функции объектов и функции морфизмов. Пусть A и B категории, тогда функтор F : A -> B сопоставляет объекту A из A объект B из B и морфизму объектов f : A -> A' из A морфизм объектов g : B -> B' из B. Функторы обязаны сохранять тождественные морфизмы, т.е., если idA : A -> A, то F(idA) = idB (а след-но прицепом и изоморфизмы). Ещё они схороняют композицию морфизмов.
Изи.
>>346535
#475 #346519
>>346451

> неубывающие отображения множеств


> n->(n-1)


Я не понял.
Но вообще твоя категория - это если просто повыкидывать из категории каноничных пространств все морфизмы, кроме одного для каждых двух пространств так чтобы это осталось категорией. У такой категории изоморфизмов, конечно, нет, но при чём тут она?
#476 #346527
Бляяяять в учебнике Зорича 2016 года опять эти ебаные окрестности, как же бомбит от этого
>>346528
#477 #346528
>>346527

>окрестности


А что должно быть?
>>346529
#478 #346529
>>346528
гамологии)
#479 #346535
>>346507

>Морфизм - это стрелка между объектами категории


Я стрелку только в смысле отображения знаю. Морфизм - это отображение?

>Функтор - морфизм целых категорий.


Звучит, вроде бы, понятно, но очень уж туманно. Где взять разжеванные для даунов примеры?
11 Кб, 856x111
51 Кб, 874x754
#480 #346537
Посаны, пикрелейтед задача сложная или я тупой? Это из Львовского. На 2м пике мое решение, решал 3 часа. Как надо было использовать тот факт, что ln вогнута и красиво решить?
>>346542>>346584
#481 #346540
Эй, математики, хочу такое спросить. Вот возьмем всю математику. Теперь удалим из неё все обозначения Бурбаки. Потом все обозначения теории категорий, Hott, в общем, любой формализм. Что-то остается от математики? Если да, вы можете это назвать?
#482 #346542
>>346537
Оформил решение в техе - сразу видно что ты продвинутый малый. Следующий шаг - верифицируешь решение в петухе (Coq) - и никаких вопросов не останется. А Львовский со своими выпуклостями пусть на хуе крутится.
>>346546
#483 #346546
>>346542
Петух это, конечно, смешно, но вопрос-то был не об этом. Суть упражнения была в какой-то фишке, которую я не понял.
#484 #346557
В английской Википедии нормальные статьи по математике? Их там действительно много, мне интересно, можно ли по ним же изучать материал?
Вот, например, статья по векторным пространствам: https://en.wikipedia.org/wiki/Vector_space
>>346559>>346564
#485 #346559
>>346557
нет нельзя.
>>346564>>346651
#486 #346564
>>346557
>>346559
Но написано лучше чем в русской.
0 Кб, 146x22
51 Кб, 250x250
#487 #346570
Посоны, вы тут, судя по названию, все умные. Расскажите мне, зачем нужны линейные, квадратные и т.п. уравнения. Что было понятно, такие как на пикче. Я смутно помню, что в классе 5-м мы такие решали просто тысячами, там еще были разновидности со скобочками. А еще были с дробями, это вообще пушка, я чуть с ума не сошел от них. Так вот, зачем это все было нужно? Я тогда у учительницы постеснялся спросить, это все имеет какой-то практический смысл или просто игры ума и все такое?
>>346571>>346704
#488 #346571
>>346570

>это все имеет какой-то практический смысл


Разумеется. Нахождение корней уравнений это база ВСЕГО.

Вот пример. Допустим, мы знаем уравнение изменение координаты какого-то тела с течением времени:
x = x0 + v0t + at^2/2
где x0 - начальная координата тела, v0 - начальная скорость тела, a - ускорение, они нам заданы.
Решая квадратное уравнение, мы можем найти в какой момент времени тело будет в точке x.

К примеру, 5 + 2t + 3t^2 = 6 - в наших терминах будет означать, что тело начало движение в точке с координатой x0 = 5 м, начальной скоростью v0 = 2 м/с и ускорением 6 м/с^2. Нам интересно, в какой момент времени тело окажется в точке x=6. Решая это уравнение, находим, что t=1/3
>>346575
#489 #346575
>>346571
Прибавлять скорость к ускорению. Физика так не работает, лол.
>>346577
#490 #346577
>>346575

>Прибавлять скорость к ускорению. Физика так не работает, лол.


Бессмысленный набор текста, что тебя беспокоит?
>>346580
#491 #346580
>>346577

>что тебя беспокоит?


Что скоро может третья мировая начаться, что цены на бананы в три раза поднялись, а годных бананов и вообще не найти стало, что очки новые покупать надо, а я не знаю где и какой марки, что я до сих пор формализм Ламбека не освоил, и прочее по мелочи.
>>346587
#492 #346584
>>346537
Ничего, что применение ln дает неравенство (третья формула в решении), которое по определению означает вогнутость?
>>346703
6079 Кб, Webm
#493 #346587
59 Кб, 807x646
#494 #346588
28 или 24
>>346594>>346599
#495 #346594
>>346588
В точности 28.
Из внутренней точки, являющейся общим концом нарисованных отрезков, проведём по отрезку в каждую из вершин квадрата. Дальше вспомним, что площади треугольников одной высоты на равных основаниях равны. Дальше элементарно.
>>346595
30 Кб, 541x320
#496 #346595
>>346594
А здесь почему неправильно
>>346596>>346597
#497 #346596
>>346595
Фигуры делятся не пополам.
>>346597
#498 #346597
>>346596
>>346595
Вы блядь серьёзно? Это же ёбаная трапеция. Хуле вы не применяете соответствующие формулы?
>>346598
#499 #346598
>>346597
Где ты увидел трапецию, наркоман?
>>346600
156 Кб, 1600x1538
#500 #346599
>>346588
Разве BC не равно AD
#501 #346600
>>346598
Обычные, сука, прямоугольные трапеции!

Это же, ёбта, математики тред. Хуле тут все такие тупые? Хотя что это я, это же математики тред.
>>346602
#502 #346602
>>346600
У трапеций оба основания должны быть параллельны. Ты тут параллельность видишь?
>>346615
#503 #346615
>>346602
Формулу можно использовать всё равно.
#504 #346651
>>346559
Почему?
#505 #346680
Есть где-то инфа, как выводили все табличные интегралы? Срочно надо
>>346702
#506 #346702
>>346680
Таблица производных, в которых дана функция y и для неё y'. Интеграл берется от f(x) с учетом, что f(x)=y' и вычисляется у.
>>346705
#507 #346703
>>346584
Почему оно означает вогнутость?
#508 #346704
>>346570
Тебя в un забанили что ли или в гугле?
15 Кб, 1080x210
#509 #346705
>>346702
Если с первыми еще все ясно, то как получился пикрел?
>>346796
Перекатs #510 #346774
#511 #346796
>>346705

>то как получился пикрел?


Ну можно к радиальным координатам перейти, а потом обратно.
Тред утонул или удален.
Это копия, сохраненная 18 апреля 2016 года.

Скачать тред: только с превью, с превью и прикрепленными файлами.
Второй вариант может долго скачиваться. Файлы будут только в живых или недавно утонувших тредах. Подробнее

Если вам полезен архив М.Двача, пожертвуйте на оплату сервера.
« /sci/В начало тредаВеб-версияНастройки
/a//b//mu//s//vg/Все доски